Bac STI2D Nouvelle-Calédonie 15 novembre 2016

Exercice 1 5 points


Suites

 

  • La politique communautaire de gestion des déchets et ses déclinaisons françaises sont définies par de nombreuses directives, dont la portée varie. Certaines ont une portée générale et d'autres concernent certaines catégories de déchets spécifiques.
  • Le Projet de Plan national de prévention des déchets $2010-2020$ concerne les déchets ménagers et assimilés (DMA). L'objectif proposé par ce projet est une réduction annuelle de 7 % des DMA produits par habitant entre $2010$ et $2020$.
  • Les DMA produits en France ont été de $590$ kg par habitant en $2011$ et de $570$ kg par habitant en $2013$.

Source ADEME

Partie A :


La réduction des DMA produits entre 2011 et 2013 atteint-elle l'objectif fixé par le Projet de Plan national de prévention des déchets ?

Partie B :


On considère que les objectifs du plan national de prévention des déchets sont atteints à partir de 2013. On modélise par une suite $\left(u_n\right)$ la quantité de DMA produits en kg par habitant, le terme $u_n$ correspondant à l'année $(2013 + n)$. Ainsi $u_0 = 570$.

  1. Calculer $u_1$.
  2. Quelle est la quantité de DMA produits, arrondie au kg par habitant, en 2015 ?
  3. Déterminer la nature de la suite $\left(u_n\right)$ puis exprimer $u_n$ en fonction de $n$.

PartieC :


On considère l'algorithme ci-dessous : $$\begin{array}{|ll|}\hline \text{Variables } &\\ &n : \text{ un nombre entier naturel}\\ &q : \text{ un nombre réel }\\ &U : \text{ un nombre réel }\\ \text{Entrée} &\\ &\text{ Saisir }n\\ \text{ Initialisation } &\\ &\text{ Affecter à } u \text{ la valeur }570\\ &\text{ Affecter à } q \text{la valeur } 0,93\\ \text{ Traitement } &\\ &\text{ Répéter } n \text{ fois } \\ &\hspace{0.5cm} \text{ Affecter à } u \text{ la valeur } u \times q\\ \text{Sortie } &\\ &\text{ Afficher } u\\ \hline \end{array}$$

  1. On entre dans l'algorithme la valeur $n = 4$. Faire fonctionner cet algorithme pour compléter les cases non grisées du tableau suivant, que l'on recopiera (on donnera des valeurs arrondies au kg près par habitant). $$ \begin{array}{ |c|c|c|c| }\hline & n & q & u \\ \hline \text{ Entrées et initialisation }& 4 &0,93 &570\\ \hline 1 er \text{ passage dans la boucle de l'algorithme }&&&\\ \hline 2 e \text{ passage dans la boucle de l'algorithme} &&&\\ \hline 3 e \text{ passage dans la boucle de l'algorithme} &&&\\ \hline 4 e \text{ passage dans la boucle de l'algorithme} &&&\\ \hline \end{array} $$
  2. Interpréter la valeur de $u$ obtenue au quatrième passage dans la boucle de l'algorithme.
  3. Quel sera le pourcentage de réduction des DMA produits entre 2013 et 2017 si l'objectif du Projet de Plan national de prévention des déchets est atteint chaque année?
  4. Quelle devrait être la quantité de DMA produits en 2020 pour atteindre l'objectif fixé par le Projet de Plan national de prévention des déchets?

 


Correction de l'exercice 1 (5 points)


Suites

  • La politique communautaire de gestion des déchets et ses déclinaisons françaises sont définies par de nombreuses directives, dont la portée varie. Certaines ont une portée générale et d'autres concernent certaines catégories de déchets spécifiques.
  • Le Projet de Plan national de prévention des déchets $2010-2020$ concerne les déchets ménagers et assimilés (DMA). L'objectif proposé par ce projet est une réduction annuelle de 7 % des DMA produits par habitant entre $2010$ et $2020$.
  • Les DMA produits en France ont été de $590$ kg par habitant en $2011$ et de $570$ kg par habitant en $2013$.

Source ADEME

Partie A :


La réduction des DMA produits entre 2011 et 2013 atteint-elle l'objectif fixé par le Projet de Plan national de prévention des déchets ?
  • méthode 1: Le coefficient multiplicateur associé à une baisse de 7 % est $1-\frac{7}{100}=0,93$.
    Avec une réduction annuelle de 7 % des DMA, la quantité de DMA par habitant en 2013 devrait être de : $590\times 0,932=510,291$
    Les DMA produits en France ayant été de 570 kg par habitant en 2013, l'objectif fixé par le Projet de Plan national de prévention des déchets n'est pas atteint en 2013.
  • méthode 2 : Calculons le pourcentage d'évolution annuel moyen des DMA produits en France entre 2011 et 2013 :
    $590\times (1+\frac{t}{100})^2=570\iff 1+\frac{t}{100} =\left(\frac{570}{590}\right)^\frac{1}{2}\iff t≈-1,7$
    Avec une réduction annuelle d'environ 1,7 % des DMA produits par habitant entre 2011 et 2013, l'objectif fixé par le Projet de Plan national de prévention des déchets n'est pas atteint en 2013.

Partie B :


On considère que les objectifs du plan national de prévention des déchets sont atteints à partir de 2013. On modélise par une suite $\left(u_n\right)$ la quantité de DMA produits en kg par habitant, le terme $u_n$ correspondant à l'année $(2013 + n)$. Ainsi $u_0 = 570$.

  1. Calculer $u_1$.
  2. $u_1= 570\times \left(1-\frac{7}{100}\right)= 510,291$
    $u1=530,1.$
  3. Quelle est la quantité de DMA produits, arrondie au kg par habitant, en 2015 ?
  4. $u_2=530,1\times 0,93=492,993$
    Les DMA produits en France ont été d'environ 493 kg par habitant en 2015.
  5. Déterminer la nature de la suite $\left(u_n\right)$ puis exprimer $u_n$ en fonction de $n$.
  6. Pour tout entier $n$ on a $u_{n+1}=0,93\times u_n$
    donc $(u_n)$ est une suite géométrique de raison $q=0,93$.
    $(u_n)$ est une suite géométrique de raison $q=0,93$ et de premier terme $u_0=570$
    donc pour tout entier $n$ on a $u_n=570\times 0,93^n$.

PartieC :


On considère l'algorithme ci-dessous : $$\begin{array}{|ll|}\hline \text{Variables } &\\ &n : \text{ un nombre entier naturel}\\ &q : \text{ un nombre réel }\\ &U : \text{ un nombre réel }\\ \text{Entrée} &\\ &\text{ Saisir }n\\ \text{ Initialisation } &\\ &\text{ Affecter à } u \text{ la valeur }570\\ &\text{ Affecter à } q \text{la valeur } 0,93\\ \text{ Traitement } &\\ &\text{ Répéter } n \text{ fois } \\ &\hspace{0.5cm} \text{ Affecter à } u \text{ la valeur } u \times q\\ \text{Sortie } &\\ &\text{ Afficher } u\\ \hline \end{array}$$

  1. On entre dans l'algorithme la valeur $n = 4$. Faire fonctionner cet algorithme pour compléter les cases non grisées du tableau suivant, que l'on recopiera (on donnera des valeurs arrondies au kg près par habitant).
  2. $$ \begin{array}{ |c|c|c|c| }\hline & n & q & u \\ \hline \text{ Entrées et initialisation }& 4 &0,93 &570\\ \hline 1 er \text{ passage dans la boucle de l'algorithme }&&& 530\\ \hline 2 e \text{ passage dans la boucle de l'algorithme} &&& 493\\ \hline 3 e \text{ passage dans la boucle de l'algorithme} &&&458\\ \hline 4 e \text{ passage dans la boucle de l'algorithme} &&& 426\\ \hline \end{array} $$
  3. Interpréter la valeur de $u$ obtenue au quatrième passage dans la boucle de l'algorithme.
  4. La quantité de DMA produits en 2017 devrait être d'environ 426 kg par habitant.
  5. Quel sera le pourcentage de réduction des DMA produits entre 2013 et 2017 si l'objectif du Projet de Plan national de prévention des déchets est atteint chaque année?
  6. $0,93^4\approx 0,748$ ce qui correspond à une réduction d'environ 25,2 %.( 1-0,748= 0,252)
  7. Quelle devrait être la quantité de DMA produits en 2020 pour atteindre l'objectif fixé par le Projet de Plan national de prévention des déchets?
  8. $u_7=570\times 0,93^7\approx 343$ La quantité de DMA produits en 2020 devrait être d'environ 343 kg par habitant.

Exercice 2 5 points


QCM


Pour chacune des cinq affirmations suivantes, indiquer si elle est vraie ou fausse en justifiant la réponse.
Il est attribué un point par réponse exacte correctement justifiée.
Une réponse non justifiée n'est pas prise en compte.

  1. Considérons les deux nombres complexes $z_1 = \sqrt{2}\text{e}^{\text{i}\frac{3\pi}{4}}$ et $z_2 = 1 - \text{i}\sqrt{3}$ où i est le nombre complexe de module 1 et d'argument $\frac{\pi}{2}$.
    Affirmation 1 : Le produit $z_1 \times z_2$ est égal à $2\sqrt{2}\text{e}^{\text{i}\frac{5\pi}{12}}$.
  2. Affirmation 2 : La solution $f$ de l'équation différentielle $y" + 4y = 0$ qui vérifie $f(0) = - 1$ et $f'(0) = 2$ admet comme représentation graphique :
  3. Affirmation 3 : La solution de l'équation $\ln(x + 3) = 5$ est $\text{e}^5 - 3$.
  4. La durée de vie en heures d'un certain type d'ampoules électriques est modélisée par une variable aléatoire qui suit la loi exponentielle de paramètre $\lambda = 0,000125 \: \left(\text{exprimé en } \:h\right)$.
    Affirmation 4 :
    En moyenne, la durée de vie d'une ampoule est 1250 h.
  5. Affirmation 5 : la fonction $F(x) = x\ln x - x + 2$ est une primitive de la fonction $f(x) = \ln x$ sur l'intervalle $]0~;~+\infty[$.

 


Correction de l'exercice 2 (5 points)


QCM
Pour chacune des cinq affirmations suivantes, indiquer si elle est vraie ou fausse en justifiant la réponse.
Il est attribué un point par réponse exacte correctement justifiée.
Une réponse non justifiée n'est pas prise en compte.

  1. Considérons les deux nombres complexes $z_1 = \sqrt{2}\text{e}^{\text{i}\frac{3\pi}{4}}$ et $z_2 = 1 - \text{i}\sqrt{3}$ où i est le nombre complexe de module 1 et d'argument $\frac{\pi}{2}$.
    Affirmation 1 : Le produit $z_1 \times z_2$ est égal à $2\sqrt{2}\text{e}^{\text{i}\frac{5\pi}{12}}$.
  2. Déterminons la forme exponentielle du nombre complexe $z_2=1-\text{i}\sqrt 3$ : $$\begin{array}{cc} \text{ Module} & \text{ Argument} \\ \begin{array}{rl|rl} |z |&=\sqrt{a^2+b^2} \\ & =\sqrt{ 1^2+\sqrt{3}^2}\\ &=\sqrt 4\\ &= 2 \end{array}& \left\lbrace \begin{array}{l} \cos \theta=\frac{a}{r}~=\frac{1}{2}\\ ~\sin \theta=\frac{b}{r}~=-\frac{\sqrt 3}{2} \end{array} \right.\\ &\text{ Donc } \theta = - \frac{\pi}{3} \text{ convient } \end{array}$$ $$z= 1 - \text{i}\sqrt 3= 2\left(\cos\left(-\frac{\pi}{3} \right) +\text{i}\sin \left(-\frac{\pi}{3} \right) \right)= 2 \text{e}^{-\text{i}\frac{\pi}{3}}$$ $$\begin{array}{rl} z_1\times z_2 &= \sqrt{2}\text{e}^{\text{i}\frac{3\pi}{4}} \times 2 \text{e}^{-\text{i}\frac{\pi}{3}} \\ & =\sqrt{2}\text{e}^{\text{i}\frac{9\pi}{12}}\times 2 \text{e}^{-\text{i}\frac{4\pi}{12}} \\ &= 2\sqrt{2}\text{e}^{\text{i}\frac{5\pi}{12}} \end{array}$$ $$\text{Le produit } z_1 \times z_2 \text{ est égal à } 2\sqrt{2}\text{e}^{\text{i}\frac{5\pi}{12}}$$
    L'affirmation 1 est vraie .
  3. Affirmation 2 : La solution $f$ de l'équation différentielle $y" + 4y = 0$ qui vérifie $f(0) = - 1$ et $f'(0) = 2$ admet comme représentation graphique :
  4. Les solutions de l'équation différentielle $y"+4 y=0$ ( du type $y"+\omega ^2 y=0$ avec $\omega =2$ sont les fonctions $f$ de la forme $f:x\mapsto A\cos \left(2x \right)+B\sin \left(2 x \right)$
    • Or $f(0)= -1 \iff A\cos \left(0 \right)+B\sin \left(0 \right) = -1\iff A= -1$
    • Comme $ f(x)= A\cos \left(2x \right)+B\sin \left(2 x \right)$ d'où on déduit $f'(x) =-2A\sin \left(2x \right)+2B\cos \left(2 x \right)$ et
    • $f'0)= 2\iff 2B = 2 \iff B=1 $
    • Ainsi, la solution $f$ de l'équation différentielle $y"+4 y=0$ qui vérifie $f(0)= -1$ et $f'(0)= 2$, est la fonction définie par $f(x)= -\cos \left(2x \right)+\sin \left(2 x \right)$.
    • Comme pour tout réel $x$ on a $-1\leq \sin 2x\leq 1$ et $-1\leq \cos 2x\leq 1$, on en déduit que $-2\leq f(x)\leq 2$.
    L'affirmation 2 est fausse .
  5. Affirmation 3 : La solution de l'équation $\ln(x + 3) = 5$ est $\text{e}^5 - 3$.
  6. $$\begin{array}{rl} \ln(x + 3) = 5& \iff\text{ e} ^{\ln(x + 3)} = \text{ e} ^ 5 \\ & \iff x + 3 = \text{ e} ^ 5 \\ & \iff x = \text{ e} ^ 5 -3\\ \end{array}$$ L'affirmation 3 est vraie .
  7. La durée de vie en heures d'un certain type d'ampoules électriques est modélisée par une variable aléatoire qui suit la loi exponentielle de paramètre $\lambda = 0,000125 \: \left(\text{exprimé en } \:h\right)$.
    Affirmation 4 :
    En moyenne, la durée de vie d'une ampoule est 1250 h.
  8. L'espérance de la variable aléatoire $X$ qui suit la loi exponentielle de paramètre $\lambda =0,000125$ est : $$E(X)=\dfrac{1}{\lambda}= \dfrac{1}{0,000125} =8000$$ L'affirmation 4 est fausse .
  9. Affirmation 5 : la fonction $F(x) = x\ln x - x + 2$ est une primitive de la fonction $f(x) = \ln x$ sur l'intervalle $]0~;~+\infty[$.
  10. $F$ est dérivable comme somme de deux fonctions dérivables.On écrit $F(x) = x\ln x - x + 2= x\left ( \ln x -1\right ) + 2 $ $F=u v ,$ d'où $F'=u'v+v'u $ avec pour tout réel $x$, dans $ ]0~;~+\infty[$ : $\left\{ \begin{array}{l} u(x)~ = x\\ v(x)~ = \ln x -1\end{array}\right.$ ainsi : $\left\{ \begin{array}{l} u'(x)~ = 1\\ v'(x)~ = \dfrac{1}{x}\end{array}\right.$ $$ \begin{array}{cl} F'(x)& = 1\times (\ln x -1) + \times x \\ & = \ln x + 1 -1\\ &= \ln x = f(x) \end{array} $$ L'affirmation 5 est vraie .

 


Exercice 3 4 points


Equation différentielle


Le bassin d'une piscine municipale a une capacité de 600000 litres d'eau. Afin de respecter les normes d'hygiène et de sécurité, 30000 litres d'eau de la piscine sont renouvelés chaque heure et le taux de chlore maximum autorisé est de $0,25$ mg/L.
Un soir après la fermeture de la piscine, alors que le taux de chlore est indétectable, $1$ kg de chlore est déversé par erreur dans le bassin à $20$  h.
Le directeur de la piscine souhaiterait savoir quand il pourra ouvrir à nouveau la piscine au public. On modélise la concentration massique du chlore présent dans la piscine par une fonction $f$. Lorsque $t$ désigne le temps écoulé depuis l'accident, exprimé en heures, $f(t)$ représente la concentration massique du chlore présent dans la piscine en milligrammes par litre. On admet que la fonction $f$ est solution de l'équation différentielle (E) : \[y' + 0,05 y = 0\quad \text{où }\: y \text{ désigne une fonction de la variable } t.\]

    1. Résoudre l'équation différentielle (E).
    2. Que vaut $f(0)$ ? En déduire une expression de $f(t)$ sur $[0~;~+ \infty[$.
  1. On admet que $f$ est définie sur $[0~;~+ \infty[$ par $f(t) = \dfrac{5}{3} \times \text{e}^{- 0,05t}$. À quel moment la piscine pourra-t-elle ouvrir de nouveau au public ?

Correction de l'exercice 3 (4 points)


Equation différentielle


Le bassin d'une piscine municipale a une capacité de 600000 litres d'eau. Afin de respecter les normes d'hygiène et de sécurité, 30000 litres d'eau de la piscine sont renouvelés chaque heure et le taux de chlore maximum autorisé est de $0,25$ mg/L.
Un soir après la fermeture de la piscine, alors que le taux de chlore est indétectable, $1$ kg de chlore est déversé par erreur dans le bassin à $20$  h.
Le directeur de la piscine souhaiterait savoir quand il pourra ouvrir à nouveau la piscine au public. On modélise la concentration massique du chlore présent dans la piscine par une fonction $f$. Lorsque $t$ désigne le temps écoulé depuis l'accident, exprimé en heures, $f(t)$ représente la concentration massique du chlore présent dans la piscine en milligrammes par litre. On admet que la fonction $f$ est solution de l'équation différentielle (E) : \[y' + 0,05 y = 0\quad \text{où }\: y \text{ désigne une fonction de la variable } t.\]

    1. Résoudre l'équation différentielle (E).
    2. Les solutions de l'équation différentielle (E) : $y'+0,05 y=0$ , qui se met sous la forme $y'= - 0, 05 y$ ( type $y'=ay$ où $a=-0,05$) sont les fonctions $f$ de la forme $f:t\mapsto k e^{-0,05 t} $ où $k$ est un réel .
    3. Que vaut $f(0)$ ? En déduire une expression de $f(t)$ sur $[0~;~+ \infty[$.
    4. Au moment de l'accident, la concentration massique du chlore présent dans la piscine en milligrammes par litre est : $ \dfrac{10^6}{600000}=\dfrac{5}{3}$ Ainsi, $f(0) = \dfrac{5}{3}$ d'où $ k e^0=\dfrac{5}{3}$ soit $k=\dfrac{5}{3}$ La fonction $f$ est définie pour tout réel $t$ de l'intervalle $[0~;~+\infty[$ par $f(t)= \dfrac{5}{3} e^{-0,05 t} $.
  1. On admet que $f$ est définie sur $[0~;~+ \infty[$ par $f(t) = \dfrac{5}{3} \times \text{e}^{- 0,05t}$. À quel moment la piscine pourra-t-elle ouvrir de nouveau au public ?
  2. $$\begin{array}{rll} \dfrac{5}{3}\times e^{-0,05 t} \leq 0, 25& \iff e^{-0,05 t}\leq 0, 25\times \dfrac{3}{5} & \text{ on multiplie par } \dfrac{3}{5}> 0 \\ & \iff e^{-0,05 t}\leq \dfrac{1}{4}\times \dfrac{3}{5} & \\ & \iff e^{-0,05 t}\leq \dfrac{3}{20} & \\ & \iff \ln\left ( e^{-0,05 t}\right )\leq \ln\left ( \dfrac{3}{20}\right ) &\text{ on applique } \ln \\ &\iff -0,05 t \leq \ln\left ( \dfrac{3}{20}\right ) & \\ &\iff t\geq -\dfrac{ \ln\left ( \dfrac{3}{20}\right )}{0,05}& \text{ on divise par } -0,05< 0 \\ &\iff t\geq -20 \ln\left ( \dfrac{3}{20}\right ) & \end{array}$$ Comme $ -20 \ln\left ( \dfrac{3}{20}\right ) \approx 37,94$ , on en déduit que : La piscine pourra être ouverte au public 38 heures après le moment de l'accident.

Exercice 4  5 points


Probabilités


Les deux parties de cet exercice peuvent être traitées de manière indépendante. Dans l'ensemble de l'exercice, les résultats seront arrondis à $10^{-3}$ près. Une usine fabrique des batteries au lithium-ion pour des vélos électriques. Le cahier des charges indique qu'une batterie mesure 15 cm de large. Lors de la fabrication, on modélise la largeur des batteries par une variable aléatoire X qui suit une loi normale de moyenne $\mu = 15$ et d'écart-type $\sigma = 0,02$. L'objectif de cet exercice est d'analyser la qualité de la production dans cette usine.

Partie A


Une batterie est jugée conforme lorsque sa largeur, exprimée en centimètres, appartient à l'intervalle [14,95 ; 15,05].

  1. Calculer la probabilité qu'une batterie prélevée au hasard dans la production soit non conforme. L'usine vend ses batteries au lithium-ion par lots de 2000 aux fabricants de vélos électriques. En moyenne, chaque lot de 2000 batteries en contient $24$ non conformes. On note $p$ la probabilité qu'une batterie soit non conforme. On prélève au hasard 2000 batteries dans la production. La production est assez importante pour que l'on puisse assimiler ce prélèvement à un tirage aléatoire avec remise. On modélise le nombre de batteries non conformes dans un lot de 2000 par une variable aléatoire $Y$.
  2. Quelle loi suit la variable aléatoire $Y$ ? Préciser ses paramètres.
  3. Calculer la probabilité qu'il y ait au moins $30$ batteries non conformes dans un lot de 2000 batteries.

 

Partie B


Dans le cadre d'un fonctionnement correct des machines de la chaîne de production, on admet que la proportion $p$ de batteries non conformes est 1,2 %. Le responsable de l'usine affirme qu'il ne vend pas de lot de 2000 batteries qui en contienne plus de $40$ non conformes. Quelle est la fiabilité de cette affirmation ? Justifier.

 


Exercice 4 5 points


Probabilités


Les deux parties de cet exercice peuvent être traitées de manière indépendante. Dans l'ensemble de l'exercice, les résultats seront arrondis à $10^{-3}$ près. Une usine fabrique des batteries au lithium-ion pour des vélos électriques. Le cahier des charges indique qu'une batterie mesure 15 cm de large. Lors de la fabrication, on modélise la largeur des batteries par une variable aléatoire X qui suit une loi normale de moyenne $\mu = 15$ et d'écart-type $\sigma = 0,02$. L'objectif de cet exercice est d'analyser la qualité de la production dans cette usine.

Partie A


Une batterie est jugée conforme lorsque sa largeur, exprimée en centimètres, appartient à l'intervalle [14,95 ; 15,05].

  1. Calculer la probabilité qu'une batterie prélevée au hasard dans la production soit non conforme. L'usine vend ses batteries au lithium-ion par lots de 2000 aux fabricants de vélos électriques. En moyenne, chaque lot de 2000 batteries en contient $24$ non conformes. On note $p$ la probabilité qu'une batterie soit non conforme. On prélève au hasard 2000 batteries dans la production. La production est assez importante pour que l'on puisse assimiler ce prélèvement à un tirage aléatoire avec remise. On modélise le nombre de batteries non conformes dans un lot de 2000 par une variable aléatoire $Y$.
  2. Une batterie est non conforme lorsque sa largeur, exprimée en centimètres, n'appartient pas à l'intervalle $[14,95;15,05]$. $$P(X\notin [14,95;15,05]) =1-P(14,95\leq X \leq 15,05) \approx 0,012$$ On détaille un peu :

    2ND DISTR 2NORMALFRép( \1 , \2,\3,\4)EXE
    Avec une calculatrice de type TI

    $$NormalFR\text{é}p(\1,\2,\3,\4) \approx \5$$

    $$P(\1 \leq \6 \leq \2)\approx \5 \text{ à } 10^{-\7} \text{ près.}$$

     

    Arrondie au millième près, la probabilité qu'une batterie prélevée au hasard dans la production soit non conforme est 0,012.
  3. Quelle loi suit la variable aléatoire $Y$ ? Préciser ses paramètres.
  4. Calculer la probabilité qu'il y ait au moins $30$ batteries non conformes dans un lot de 2000 batteries.
  5. La production est assez importante pour que l'on puisse assimiler le prélèvement de 2000 batteries à un tirage aléatoire avec remise donc :
    La variable aléatoire $Y$ suit la loi binomiale de paramètres $n=2000$ et $p=\dfrac{24}{2000}=0,012$.

 

Partie B


Dans le cadre d'un fonctionnement correct des machines de la chaîne de production, on admet que la proportion $p$ de batteries non conformes est 1,2 %. Le responsable de l'usine affirme qu'il ne vend pas de lot de 2000 batteries qui en contienne plus de $40$ non conformes. Quelle est la fiabilité de cette affirmation ? Justifier.

La proportion $p$ est égale à  $\1$. La taille  $n$  de l'échantillon considéré est égale à  $\2.$
Comme  $ n =\2$ ,   $n \times p  $=\3  et $n\times (1-p)=\4,$ les conditions d'utilisation d'un intervalle de fluctuation asymptotique sont réunies.

En effet on a bien : $$n \geq 30\;;\; n \times p \geq 5 \text{ et } n\times (1-p) \geq 5$$


L'intervalle de fluctuation asymptotique au seuil de  $95\% $  est : $$I_{\2} = \left[\1 - 1,96\sqrt{\dfrac{\1\times \5}{\2}}~;~\1 + 1,96\sqrt{\dfrac{\1\times \5}{\2}} \right]$$ 

$$I_{2000}\approx[0,007; 0,017]$$ Soit un intervalle de fluctuation avec un niveau de confiance de 95 % du nombre de batteries non conformes dans un échantillon de taille 2000 : $I=[0,007\times 20000; 017\times 2000] =[14; 34]$

Avec un niveau de confiance de 95 % on accepte l'affirmation du responsable de l'usine.

 

  • Vues: 9603

Baccalauréat STI 2D/STL spécialité SPCL Métropole- La Réunion 8 septembre 2016

 

Exercice 1 6 points


Fonction exponentielle


Dans cet exercice, la température est exprimée en degrés Celsius (°C) et le temps $t$ est exprimé en minutes. Dans une entreprise de fabrication de pièces métalliques, un ouvrier doit manipuler des plaques chaudes pendant une dizaine de secondes. À la sortie du four, les plaques sont à une température de 30 °C et disposées dans une pièce dont la température ambiante est maintenue à 26 °C par un système de ventilation. La commission de sécurité prescrit qu'avec les gants actuels, l'ouvrier doit attendre 10 minutes pour manipuler les plaques à leur sortie du four. Afin de réduire ce délai d'attente, le directeur s'interroge sur l'achat de nouveaux gants dont les caractéristiques techniques établies par la commission de sécurité sont les suivantes :

  • Sans couture.
  • Très doux et confortables.
  • Température maximale d'utilisation : 240 °C.
  1. Dans cette question, on ne demande pas de justification.
    1. Quelle est, à la sortie du four, la température des plaques ?
    2. Comment varie, à la sortie du four, la température des plaques au cours du temps ?
    3. Vers quelle valeur la température des plaques devrait-elle se stabiliser ?
  2. La température d'une plaque depuis sa sortie du four, est modélisée en fonction du temps $t$, exprimé en minutes, par une fonction $g$. On admet que cette fonction $g$ est définie sur l'intervalle $[0~;~+\infty [$ par $g(t) = 274 e^{at} + 26$ où $a$ est un nombre réel.
    1. Calculer $g(0)$. Ce résultat est-il conforme aux données ?
    2. D'après la question 1, quel doit être le signe du nombre réel $a$ ?
    3. On sait que 3 minutes après sa sortie du four la température de la plaque, arrondie à l'unité, est de 262 °C. Montrer que la valeur approchée à $10^{-2}$ près du coefficient $a$ est $-0,05$.
  3. Dans cette question on considère que, pour tout nombre réel $t$ de l'intervalle $[0 ; +\infty [$ : \[ g(t)=274 e^{-0,05t}+26 .\]
    1. Avec les gants actuellement utilisés, à quelle température l'ouvrier pourra-t-il manipuler les plaques après leur sortie du four, en respectant les caractéristiques techniques de la commission de sécurité ?
    2. Si le directeur décidait d'équiper les ouvriers avec les nouveaux gants, quel délai d'attente minimal serait requis avant que les ouvriers puissent manipuler les plaques ?
    3. En déduire le gain de temps, en pourcentage, dû à l'utilisation de ces nouveaux gants.

Correction de l'exercice 1 (6 points)


Fonction exponentielle


Dans cet exercice, la température est exprimée en degrés Celsius (°C) et le temps $t$ est exprimé en minutes. Dans une entreprise de fabrication de pièces métalliques, un ouvrier doit manipuler des plaques chaudes pendant une dizaine de secondes. À la sortie du four, les plaques sont à une température de 30 °C et disposées dans une pièce dont la température ambiante est maintenue à 26 °C par un système de ventilation. La commission de sécurité prescrit qu'avec les gants actuels, l'ouvrier doit attendre 10 minutes pour manipuler les plaques à leur sortie du four. Afin de réduire ce délai d'attente, le directeur s'interroge sur l'achat de nouveaux gants dont les caractéristiques techniques établies par la commission de sécurité sont les suivantes :

  • Sans couture.
  • Très doux et confortables.
  • Température maximale d'utilisation : 240 °C.
  1. Dans cette question, on ne demande pas de justification.
    1. Quelle est, à la sortie du four, la température des plaques ?
    2. À la sortie du four, les plaques sont à une température de 300°C.
    3. Comment varie, à la sortie du four, la température des plaques au cours du temps ?
    4. À la sortie du four, les plaques sont disposées dans une pièce dont la température ambiante est maintenue à 26°C donc la température des plaques va baisser.
    5. Vers quelle valeur la température des plaques devrait-elle se stabiliser ?
    6. La température de la pièce est maintenue à 26°C donc la température des plaques devrait-elle se stabiliser à 26°C.
  2. La température d'une plaque depuis sa sortie du four, est modélisée en fonction du temps $t$, exprimé en minutes, par une fonction $g$. On admet que cette fonction $g$ est définie sur l'intervalle $[0~;~+\infty [$ par $g(t) = 274 e^{at} + 26$ où $a$ est un nombre réel.
    1. Calculer $g(0)$. Ce résultat est-il conforme aux données ?
    2. $$g⁡(0)=274\times e^{0}+26=300$$
    3. D'après la question 1, quel doit être le signe du nombre réel $a$ ?
    4. Comme la fonction $g$ est décroissante, le nombre réel $a$ est négatif.
    5. On sait que 3 minutes après sa sortie du four la température de la plaque, arrondie à l'unité, est de 262 °C. Montrer que la valeur approchée à $10^{-2}$ près du coefficient $a$ est $-0,05$.
    6. Le nombre réel $a$ est solution de l'équation : $$\begin{array}{rl} g(3)=262 & \iff 274 e^{3a} + 26= 262 \\ & \iff 274 e^{3a} = 236 \\ &\iff e^{3a} = \dfrac{236}{274} \\ &\iff 3a =\ln\left (\dfrac{236}{274}\right )\\ &\iff a= \dfrac{\ln\left (\dfrac{236}{274}\right )}{3}\approx - 0, 05 \end{array}$$ La valeur approchée à $10^{-2}$ près du coefficient $a$ est $a\approx -0,05$.
  3. Dans cette question on considère que, pour tout nombre réel $t$ de l'intervalle $[0 ; +\infty [$ : \[ g(t)=274 e^{-0,05t}+26 .\]
    1. Avec les gants actuellement utilisés, à quelle température l'ouvrier pourra-t-il manipuler les plaques après leur sortie du four, en respectant les caractéristiques techniques de la commission de sécurité ?
    2. $g(10 ) = 274 e^{-0,5} + 26\approx 192$ Avec les gants actuellement utilisés, l'ouvrier pourra manipuler les plaques à la température de 192°C.
    3. Si le directeur décidait d'équiper les ouvriers avec les nouveaux gants, quel délai d'attente minimal serait requis avant que les ouvriers puissent manipuler les plaques ?
    4. Le délai d'attente $t$ est solution de l'inéquation : $$\begin{array}{rl} g(t)\leq 240& \iff 274 e^{-0,05t} + 26\leq 240\\ & \iff 274 e^{-0,05t} \leq 214\\ & \iff e^{-0,05t} \leq \dfrac{214}{274}\\ & \iff \ln\left ( e^{-0,05t}\right ) \leq \ln\left (\dfrac{214}{274}\right )\\ & \iff -0,05t \leq \ln\left (\dfrac{214}{274}\right )\\ & \iff t \geq \dfrac{ \ln\left (\dfrac{214}{274}\right )}{-0,05}\\ & \iff t \geq 20 \ln\left (\dfrac{137}{107}\right ) \\ \end{array}$$ Comme $20 \ln\left (\dfrac{137}{107}\right ) \approx 4,943$, on en déduit que : Avec les nouveaux gants, l'ouvrier pourra manipuler les plaques au bout de 5 minutes.
    5. En déduire le gain de temps, en pourcentage, dû à l'utilisation de ces nouveaux gants.

    6. Avec les nouveaux gants, le temps d'attente est divisé par deux soit un gain de 50%.

Exercice 2 4 points


Probabilités


Les parties A et B sont indépendantes. Dans cet exercice, toutes les probabilités demandées seront arrondies à $10^{-3}$.
Une usine métallurgique fabrique des boîtes de conserve pour des entreprises spécialisées dans le conditionnement industriel de légumes. La probabilité qu'une boîte prélevée au hasard soit non conforme est 0,04. Un lot de 200 boîtes choisies au hasard est livré à une entreprise spécialisée dans le conditionnement des légumes. Le nombre de boîtes fabriquées par cette usine métallurgique est assez important pour pouvoir assimiler un tel prélèvement à un tirage avec remise de 200 boîtes.

Partie A


La variable aléatoire $X$ désigne le nombre de boîtes non conformes dans un tel lot.

  1. Justifier que la variable aléatoire $X$ suit une loi binomiale dont on précisera les paramètres.
  2. Déterminer la probabilité qu'un tel lot contienne exactement quatre boîtes non conformes.

 

Partie B


On décide d'approcher la loi binomiale suivie par $X$ par la loi normale d'espérance $\mu = 8$ et d'écart type $\sigma = 2,77$.

  1. Justifier le choix de ces paramètres.
  2. À l'aide de la loi normale ainsi définie :
    1. calculer $P(6 \leqslant X \leqslant 10)$ et interpréter le résultat trouvé ;
    2. déterminer une approximation de la probabilité qu'il y ait au maximum 4 boîtes non conformes.

 

Partie C


Dans le lot livré de 200 boîtes, on compte 11 boîtes non conformes. Le fabricant des boîtes est averti. Doit-il s'inquiéter ?
On pourra utiliser un intervalle de fluctuation.


Correction de l'exercice 2 (4 points)


Probabilités

Exercice 2 4 points


Probabilités


Les parties A et B sont indépendantes. Dans cet exercice, toutes les probabilités demandées seront arrondies à $10^{-3}$.
Une usine métallurgique fabrique des boîtes de conserve pour des entreprises spécialisées dans le conditionnement industriel de légumes. La probabilité qu'une boîte prélevée au hasard soit non conforme est 0,04. Un lot de 200 boîtes choisies au hasard est livré à une entreprise spécialisée dans le conditionnement des légumes. Le nombre de boîtes fabriquées par cette usine métallurgique est assez important pour pouvoir assimiler un tel prélèvement à un tirage avec remise de 200 boîtes.

Partie A


La variable aléatoire $X$ désigne le nombre de boîtes non conformes dans un tel lot.

  1. Justifier que la variable aléatoire $X$ suit une loi binomiale dont on précisera les paramètres.
  2. Le nombre de boîtes est assez important pour pouvoir assimiler un tel prélèvement à un tirage avec remise de 200 boîtes par conséquent, la variable aléatoire $X$ suit une loi binomiale de paramètres $n=200$ et $p=0,04$ .
  3. Déterminer la probabilité qu'un tel lot contienne exactement quatre boîtes non conformes.
  4. 2ND DISTR 0binomFdP( \1 , \2,\3)EXE
    Avec une calculatrice de type TI $binomFdP(\1,\2,\3) \approx \4$

    $$P( \5 = \3)\approx \4 \text{ à } 10^{-\6} \text{ près.}$$
    La probabilité qu'un lot contienne exactement quatre boîtes non conformes est 0,055.

 

Partie B


On décide d'approcher la loi binomiale suivie par $X$ par la loi normale d'espérance $\mu = 8$ et d'écart type $\sigma = 2,77$.

  1. Justifier le choix de ces paramètres.
  2. La variable aléatoire X suit une loi binomiale de paramètres n=200 et p=0,04 alors :
    • son espérance vaut $E(X)==n\times p= 200\times 0,04=8$ ;
    • son écart-type vaut $\sigma =\sqrt{n\times p\times q}= \sqrt{200\times 0,04\times 0,96}\approx 2,77$.
    En outre, on a $n=200, n⁢p=200\times 0,04=8$ et $n⁢(1-p)=200\times 0,96=192$.
    Donc les trois conditions $n\geq 30, n⁢p\geq 5$ et $n⁢(1-p)\geq 5$ sont réunies pour approcher la loi binomiale par la loi normale.
    La loi binomiale suivie par $X$ peut être approchée par la loi normale d'espérance $\mu=8$ et d'écart-type $\sigma=2,77$.
  3. À l'aide de la loi normale ainsi définie :
    1. calculer $P(6 \leqslant X \leqslant 10)$ et interpréter le résultat trouvé ;
    2. 2ND DISTR 2NORMALFRép( \1 , \2,\3,\4)EXE
      Avec une calculatrice de type TI

      $$NormalFR\text{é}p(\1,\2,\3,\4) \approx \5$$

      $$P(\1 \leq \6 \leq \2)\approx \5 \text{ à } 10^{-\7} \text{ près.}$$

       

      La probabilité qu'un lot contienne entre 6 et 10 boîtes non conformes est 0,53.
    3. déterminer une approximation de la probabilité qu'il y ait au maximum 4 boîtes non conformes.
    4. 2ND DISTR 2NORMALFRép( -10^(99) , \1,$\2$,$\3$)EXE
      Avec une calculatrice de type TI

      $$NormalFR\text{é}p(-10^{99},\1,\2,\3) \approx \4$$

      $$P( \5 \leq \1)\approx \4 \text{ à } 10^{-\6} \text{ près.}$$
      La probabilité qu'il y ait au maximum 4 boîtes non conformes est 0,074.

 

Partie C

 

Dans le lot livré de 200 boîtes, on compte 11 boîtes non conformes. Le fabricant des boîtes est averti. Doit-il s'inquiéter ?
On pourra utiliser un intervalle de fluctuation.

La frequence de boîtes non conformes dans l'échantillon prélevé est $f=\frac{11}{200}=0,055$.

La proportion $p$ est égale à  $\1$. La taille  $n$  de l'échantillon considéré est égale à  $\2.$
Comme  $ n =\2$ ,   $n \times p  $=\3  et $n\times (1-p)=\4,$ les conditions d'utilisation d'un intervalle de fluctuation asymptotique sont réunies.

En effet on a bien : $$n \geq 30\;;\; n \times p \geq 5 \text{ et } n\times (1-p) \geq 5$$


L'intervalle de fluctuation asymptotique au seuil de  $95\% $  est : $$I_{\2} = \left[\1 - 1,96\sqrt{\dfrac{\1\times \5}{\2}}~;~\1 + 1,96\sqrt{\dfrac{\1\times \5}{\2}} \right]$$ 

Soit avec des valeurs approchées à $10^{-3 }$ près des bornes de l'intervalle, l'intervalle de fluctuation avec un niveau de confiance de 95 % de la fréquence de boîtes non conformes dans un échantillon de taille 200 est $I=[0,012;0,068]$. La fréquence de boîtes non conformes dans l'échantillon prélevé appartient à l'intervalle de fluctuation asymptotique au seuil de 95% donc cet échantillon est représentatif de la production du fabricant.

Exercice 3 3 points


QCM


Pour chacune des 3 propositions suivantes, indiquer si elle est vraie ou fausse en justifiant la réponse. Il est attribué un point par réponse exacte correctement justifiée. Une réponse non justifiée n'est pas prise en compte. Une absence de réponse n'est pas pénalisée.Indiquer sur la copie le numéro de la proposition, la réponse correspondante et la justification.

  1. Proposition 1 : $e^{\mathrm{i}\frac{\pi}4{}}+e^{\mathrm{i}\frac{3\pi}4{}}=\mathrm{i}\sqrt{2}$.
  2. La durée de vie, en heures, d'un composant électronique est une variable aléatoire $T$ qui suit la loi exponentielle de paramètre $\lambda = 5,5 \times 10^{-4}$ et dont la fonction de densité de probabilité est représentée ci-dessous.
    Ex3 Sept 2016
    Proposition 2 : la probabilité, arrondie à 0,01 près, qu'un composant électronique pris au hasard ait une durée de vie inférieure à 1000 heures est 0,35.
  3. Proposition 3 : la valeur moyenne de la fonction $f$ définie sur l'intervalle $\left[\dfrac{\pi}{2} ; \pi\right]$ par $f (x) = \cos(x)$ est $-\dfrac{2}{\pi}$.
    On rappelle que la valeur moyenne $\mu$d'une fonction $f$ définie et continue sur un intervalle $[a ; b]$ est donnée par la formule :\[\mu = \dfrac{1}{b-a}\displaystyle\int_{a}^{b}f(x)\mathrm{d} x.\]

Correction de l'exercice 3 (3 points)


QCM


Pour chacune des 3 propositions suivantes, indiquer si elle est vraie ou fausse en justifiant la réponse. Il est attribué un point par réponse exacte correctement justifiée. Une réponse non justifiée n'est pas prise en compte. Une absence de réponse n'est pas pénalisée.Indiquer sur la copie le numéro de la proposition, la réponse correspondante et la justification.

  1. Proposition 1 : $e^{\mathrm{i}\frac{\pi}4{}}+e^{\mathrm{i}\frac{3\pi}4{}}=\mathrm{i}\sqrt{2}$.
  2. $$\begin{array}{rl} e^{\mathrm{i}\frac{\pi}4{}}+e^{\mathrm{i}\frac{3\pi}4{}}&= \cos\left ( \frac{\pi}4{} \right ) + \mathrm{i} \sin\left ( \frac{\pi}4{} \right )+ \cos\left ( \frac{3\pi}4{} \right ) + \mathrm{i} \sin\left ( 3\frac{\pi}4{} \right ) \\ &= \dfrac{\sqrt 2 }{2} + \mathrm{i} \dfrac{\sqrt 2 }{2}-\dfrac{\sqrt 2 }{2} + \mathrm{i} \dfrac{\sqrt 2 }{2}\\ &=\mathrm{i}\sqrt 2 \end{array}$$ La proposition 1 est vraie.
  3. La durée de vie, en heures, d'un composant électronique est une variable aléatoire $T$ qui suit la loi exponentielle de paramètre $\lambda = 5,5 \times 10^{-4}$ et dont la fonction de densité de probabilité est représentée ci-dessous.
    Ex3 Sept 2016
    Proposition 2 : la probabilité, arrondie à 0,01 près, qu'un composant électronique pris au hasard ait une durée de vie inférieure à 1000 heures est 0,35.
  4. La variable aléatoire $T$ suit la loi exponentielle de paramètre $\lambda =5,5\times 10^{-4}$ d'où : $$\begin{array}{rl} P(T\leq 1000 ) &= \displaystyle \int_0^{10200}\lambda\text{e}^{-\lambda t}\;\text{d}t \\ &= \left [ -\text{e}^{-\lambda t}\right ]_0^{1000}\\ &=-\text{e}^{-1000 \lambda }-\left (-\text{e}^{0}\right )\\ &= 1- \text{e}^{-1000 \times 5,5\times 10^{-4} }\\ &= 1-\text{e}^{-0,55 }\\ &\approx 0, 423 \end{array}$$ La proposition 2 est fausse.
  5. Proposition 3 : la valeur moyenne de la fonction $f$ définie sur l'intervalle $\left[\dfrac{\pi}{2} ; \pi\right]$ par $f (x) = \cos(x)$ est $-\dfrac{2}{\pi}$.
    On rappelle que la valeur moyenne $\mu$d'une fonction $f$ définie et continue sur un intervalle $[a ; b]$ est donnée par la formule :\[\mu = \dfrac{1}{b-a}\displaystyle\int_{a}^{b}f(x)\mathrm{d} x.\]
  6. La valeur moyenne de la fonction $f$ définie sur l'intervalle $\left[\dfrac{\pi}{2} ; \pi\right]$ par $f (x) = \cos(x)$ est : $$\begin{array}{rl} \mu &= \dfrac{1}{\pi -\dfrac{\pi}{2}}\displaystyle\int_{\dfrac{\pi}{2}}^{\pi }\cos x \mathrm{d} x \\ &= \left [ \sin x \right ]_{\dfrac{\pi}{2}} ^{\pi }\\ &=\dfrac{2}{\pi }\left ( \sin \pi-\sin\left (\dfrac{\pi}{2}\right )\right )\\ &= \dfrac{2}{\pi } ( 0-1)\\ &=- \dfrac{2}{\pi } \end{array}$$ La proposition 3 est vraie.

 


Exercice 4 7 points


Suites


Les parties A, B et C sont indépendantes.
Dans une municipalité, la collecte des déchets des particuliers s'effectue, depuis 2012, à l'aide de camions équipés de capteurs. Une tarification « incitative » permet aux habitations de diminuer leur facture en réduisant la masse de leurs ordures ménagères résiduelles par un choix de produits comportant moins d'emballages, une réduction du gaspillage alimentaire et un meilleur tri. Le document 1 présente la masse moyenne de déchets, en kilogrammes, collectés par année depuis 2012 et par habitation de la ville. Le document 2 présente les tarifs pratiqués en 2015 par la ville pour la collecte des ordures ménagères résiduelles (on suppose que ces tarifs resteront identiques les années suivantes).

Document 1

Années 2012 à 2015


$$\begin{array} {|c| c|c|c|c|} \hline \text{Année} &\text{2012} & \text{2013} & \text{2014} & \text{2015}\\ \hline \text{Déchets recyclables} & 261 & 275 & 289 & 305\\ \hline \text{Ordures ménagères résiduelles} & 274 & 269 & 262 & 256\\ \hline \text{Total} & 535 & 544 & 551 & 561\\ \hline \end{array}$$

Document 2

Année 2015


$$\begin{array}{|c| c|c|c|} \hline \text{Tranches } \text{tarifaires} & \text{Tranche 1} & \text{Tranche 2} & \text{Tranche 3}\\ \hline \text{Masse M en} \text{kilogrammes} & 0 \leqslant M < 100 & 100 \leqslant M < 300 & 300 \leqslant M \\ \hline \text{Forfait} & 200 € & 300 € & 420 €\\ \hline \end{array} $$

Partie A

 

  1. Commenter l'évolution de la masse moyenne des déchets collectés par habitation depuis 2012.
  2. Une famille a jeté 320 kg d'ordures ménagères résiduelles en 2015. Si elle diminue la masse de ses ordures ménagères résiduelles de 1 % par an, en quelle année changera-t-elle de tranche tarifaire ?

 

Partie B


En 2015, la municipalité comptait 10000 habitations. Dans le cadre de l'aménagement d'un nouveau quartier un constructeur garantit la livraison de 300 nouvelles habitations chaque année au 1er janvier, de 2016 à 2024. En raison de la demande, ces logements seront immédiatement occupés dès le 1er janvier. La municipalité a souscrit avec un centre d'incinération un contrat de 9 ans qui a pris effet au 1er janvier 2016. Le contrat prévoit de fortes pénalités financières dès que la masse annuelle d'ordures ménagères résiduelles à incinérer vient à dépasser 2800 tonnes. L'objectif de la municipalité est d'éviter ces pénalités.

  1. Vérifier que cet objectif ne sera pas atteint si la masse annuelle moyenne d'ordures ménagères résiduelles par habitation reste constante égale à 256 kg.
  2. Afin d'atteindre cet objectif, il convient donc de diminuer la masse moyenne d'ordures ménagères résiduelles à incinérer. La municipalité souhaite déterminer le pourcentage annuel minimal de réduction de la masse moyenne d'ordures ménagères résiduelles par habitation, pendant toute la durée du contrat. On admet que l'algorithme ci-dessous détermine ce pourcentage.
    Variables
    • $N$ : un nombre entier
    • $m$ : un nombre réel
    • $q$ : un nombre réel
    Initialisation
    • $q$ prend la valeur 1
    • $N$ prend la valeur 12700
    • $m$ prend la valeur 0,256
      Traitement
    • Tant que $N \times m \geqslant 2800$
      $q$ prend la valeur $q-0,001$
      $m$ prend la valeur $0,256 \times q^9$
    • Fin Tant que
      Sortie Afficher $\left(1-q\right) \times 100$
  3. Cet algorithme affiche 1,7.
    1. Expliquer la ligne « $N$ prend la valeur 12700 »
    2. Expliquer la ligne « $m$ prend la valeur $0,256 \times q^9$ »
  4. On considère que la masse annuelle moyenne d'ordures ménagères résiduelles par habitation va baisser chaque année de 1,7 %, à partir du 1\up{er} janvier 2016 sur une période de 9 ans. On note $u_n$ cette masse, exprimée en tonnes, pour l'année $2015 + n$ où $n$ est un entier naturel. On a donc $u_0 = 0,256$.
    1. Calculer les termes $u_1$, $u_2$ et vérifier que $u_3\approx 0,243$. Interpréter $u_3$.
    2. Quelle est la nature de la suite $\left(u_n\right)$ ?
    3. Exprimer $u_n$ en fonction de $n$.
    4. Vérifier que l'objectif fixé par la municipalité est atteint en fin de 2024.

 

Partie C


Dans cette partie, tous les résultats demandés seront arrondis à $10^{-3}$. Des contrôles sont effectués afin de vérifier le tri des déchets.

Protocole d'étude

On choisit au hasard $100$ habitations. Des personnels ont ouvert les poubelles de déchets recyclables de ces habitations afin de déterminer s'ils étaient conformes (absence de matériaux non recyclables, de cartons souillés $\ldots$).

Résultats de l'étude

Parmi ces 100 poubelles de déchets recyclables, 7 ont été jugées non conformes.

  1. Déterminer, à l'aide d'un intervalle de confiance avec un niveau de confiance de 95 %, une estimation de la proportion de poubelles de déchets recyclables qui ne sont pas conformes.
  2. La proportion de poubelles de déchets recyclables qui ne sont pas conformes est-elle nécessairement comprise dans cet intervalle de confiance ?

Exercice 4 7 points


Suites


Les parties A, B et C sont indépendantes.
Dans une municipalité, la collecte des déchets des particuliers s'effectue, depuis 2012, à l'aide de camions équipés de capteurs. Une tarification « incitative » permet aux habitations de diminuer leur facture en réduisant la masse de leurs ordures ménagères résiduelles par un choix de produits comportant moins d'emballages, une réduction du gaspillage alimentaire et un meilleur tri. Le document 1 présente la masse moyenne de déchets, en kilogrammes, collectés par année depuis 2012 et par habitation de la ville. Le document 2 présente les tarifs pratiqués en 2015 par la ville pour la collecte des ordures ménagères résiduelles (on suppose que ces tarifs resteront identiques les années suivantes).

Document 1

Années 2012 à 2015


$$\begin{array} {|c| c|c|c|c|} \hline \text{Année} &\text{2012} & \text{2013} & \text{2014} & \text{2015}\\ \hline \text{Déchets recyclables} & 261 & 275 & 289 & 305\\ \hline \text{Ordures ménagères résiduelles} & 274 & 269 & 262 & 256\\ \hline \text{Total} & 535 & 544 & 551 & 561\\ \hline \end{array}$$

Document 2

Année 2015


$$\begin{array}{|c| c|c|c|} \hline \text{Tranches } \text{tarifaires} & \text{Tranche 1} & \text{Tranche 2} & \text{Tranche 3}\\ \hline \text{Masse M en} \text{kilogrammes} & 0 \leqslant M < 100 & 100 \leqslant M < 300 & 300 \leqslant M \\ \hline \text{Forfait} & 200 € & 300 € & 420 €\\ \hline \end{array} $$

Partie A

 

  1. Commenter l'évolution de la masse moyenne des déchets collectés par habitation depuis 2012.
  2. Soit t % le pourcentage d'augmentation annuel moyen de la masse des déchets collectés par habitation de 2012 à 2015. On a : $$\begin{array}{rl} 535\times \left(1+\dfrac{t}{100} \right)^3= 561& \iff \left(1+\dfrac{t}{100} \right)^3 = \dfrac{561}{535} \\ & \left(1+\dfrac{t}{100} \right) = \left(\dfrac{561}{535} \right)^{\frac{1}{3}}\\ &\iff \dfrac{t}{100} \approx 0,0159 \end{array}$$ Depuis 2012, la masse moyenne des déchets collectés par habitation a augmenté d'envirion 1,6 % par an.
  3. Une famille a jeté 320 kg d'ordures ménagères résiduelles en 2015. Si elle diminue la masse de ses ordures ménagères résiduelles de 1 % par an, en quelle année changera-t-elle de tranche tarifaire ?
  4. Soit $n$ le nombre d'années nécessaires pour passer dans la tranche 2. $n$ est le plus petit entier solution de l'inéquation : $$\begin{array}{rll} 320\times 0,99^n< 300 & \iff 0,99^n < \dfrac{300}{320}&\\ & \iff \ln\left (0,99^n\right ) \leqslant \ln\left (\dfrac{15}{16}\right ) & \text{ car } \ln \text{ est strictement croissante } \\ && \text{ sur } ]0; +\infty[ \\ &\iff n \ln\left (0,9 \right ) \leqslant\ln\left (\dfrac{15}{16}\right ) & \text{ car } \ln\left (a^n\right )=n\ln a \\ &\iff n\geq \dfrac{\ln\left (\dfrac{15}{16}\right )}{\ln\left (0,99 \right )}& \text{ car } 0,99< 1 \text{ ainsi } \ln\left (0,9 \right )< 0\\ \end{array}$$ Comme $\dfrac{\ln\left (\dfrac{15}{16}\right )}{\ln\left (0,99 \right )}\approx 6,4$ on en déduit que $n=7$ Si cette famille diminue la masse de ses ordures ménagères résiduelles de 1 % par an, c'est en 2022 qu'elle changera de tranche tarifaire.

 

Partie B


En 2015, la municipalité comptait 10000 habitations. Dans le cadre de l'aménagement d'un nouveau quartier un constructeur garantit la livraison de 300 nouvelles habitations chaque année au 1er janvier, de 2016 à 2024. En raison de la demande, ces logements seront immédiatement occupés dès le 1er janvier. La municipalité a souscrit avec un centre d'incinération un contrat de 9 ans qui a pris effet au 1er janvier 2016. Le contrat prévoit de fortes pénalités financières dès que la masse annuelle d'ordures ménagères résiduelles à incinérer vient à dépasser 2800 tonnes. L'objectif de la municipalité est d'éviter ces pénalités.

  1. Vérifier que cet objectif ne sera pas atteint si la masse annuelle moyenne d'ordures ménagères résiduelles par habitation reste constante égale à 256 kg.
  2. Soit $a_n$ le nombre d'habitations le 1er janvier de l'année $2015+n$.
    On a $a_0=10000$ et, pour tout entier $n, a_{n+1}=a_n+300$.
    La suite $(a_n)$ est une suite arithmétique de premier terme $a_0=10000$ et de raison $r=300$. Donc pour tout entier $n$, on a $a_n=10000+300\times n$.
    La masse d'ordures ménagères résiduelles à incinérer , exprimée en tonnes, pour l'année $2015+n$ est $d_n=a_n\times 0,256$.
    Soit pour tout entier $n, d_n=10000+300\times n\times 0,256=2560+76,8\times n$
    Donc : $$\begin{array}{rl} d_n \leq 2800 &\iff 2560+76,8\times n\leq 2800 \\ &\iff 76,8\times n\leq 2800 \\ &\iff n\leq \dfrac{240}{76,8}\\ &\iff n\leq 3,125 \end{array}$$ Si la masse annuelle moyenne d'ordures ménagères résiduelles par habitation reste constante égale à 256 kg, l'objectif ne sera pas atteint dès la quatrième année c'est à dire au 1er janvier 2019.
  3. Afin d'atteindre cet objectif, il convient donc de diminuer la masse moyenne d'ordures ménagères résiduelles à incinérer. La municipalité souhaite déterminer le pourcentage annuel minimal de réduction de la masse moyenne d'ordures ménagères résiduelles par habitation, pendant toute la durée du contrat. On admet que l'algorithme ci-dessous détermine ce pourcentage.
    Variables
    • $N$ : un nombre entier
    • $m$ : un nombre réel
    • $q$ : un nombre réel
    Initialisation
    • $q$ prend la valeur 1
    • $N$ prend la valeur 12700
    • $m$ prend la valeur 0,256
      Traitement
    • Tant que $N \times m \geqslant 2800$
      $q$ prend la valeur $q-0,001$
      $m$ prend la valeur $0,256 \times q^9$
    • Fin Tant que
      Sortie Afficher $\left(1-q\right) \times 100$
  4. Cet algorithme affiche 1,7.
    1. Expliquer la ligne « $N$ prend la valeur 12700 »
    2. Le nombre d'habitations en 2024 est : $$a_9=10000+300\times 9=12700$$ $N=12700$ modélise le nombre d'habitations en 2024.
    3. Expliquer la ligne « $m$ prend la valeur $0,256 \times q^9$ »
    4. Le coefficient multiplicateur associé à une baisse annuelle de $t\; \%$ de la masse d'ordures ménagères résiduelles par habitation est : $q=1-\dfrac{t}{100}$
      Au bout de 9 ans, la masse annuelle moyenne d'ordures ménagères résiduelles par habitation, exprimée en tonnes, est donc $m=0,256\times q^9$
  5. On considère que la masse annuelle moyenne d'ordures ménagères résiduelles par habitation va baisser chaque année de 1,7\%, à partir du 1\up{er} janvier 2016 sur une période de 9 ans. On note $u_n$ cette masse, exprimée en tonnes, pour l'année $2015 + n$ où $n$ est un entier naturel. On a donc $u_0 = 0,256$.
    1. Calculer les termes $u_1$, $u_2$ et vérifier que $u_3\approx 0,243$. Interpréter $u_3$.
    2. Le coefficient multiplicateur associé à une baisse annuelle de $1,7\;\ %$ de la masse d'ordures ménagères résiduelles par habitation est : $q=1-\dfrac{1,7}{100}=0,983$ On en déduit que :
      • $u_1=0,256\times 0,983\approx 0,243$
      • $u_2=0,256\times 0,9832\approx 0,247$
      • $u_3=0,256\times 0,9833\approx 0,243$
      Fin 2018 ( ou au 1er janvier 2019), la masse annuelle moyenne d'ordures ménagères résiduelles par habitation est d'environ 243 kg.
    3. Quelle est la nature de la suite $\left(u_n\right)$ ?
    4. Pour tout entier $n, u_{n+1}=u_n\times 0,983$ donc $(u_n)$ est une suite géométrique de raison $q=0,983$.
    5. Exprimer $u_n$ en fonction de $n$.
    6. $(u_n)$ est une suite géométrique de raison $q=0,983$ et de premier terme $u_0=0,256$ donc pour tout entier $n$, on a $u_n=0,256\times 0,983^n.$
    7. Vérifier que l'objectif fixé par la municipalité est atteint en fin de 2024.
    8. À la fin de l'année 2024, le nombre d'habitations est de 12700 et la masse annuelle moyenne d'ordures ménagères résiduelles par habitation est $m=0,256\times 0,9839$.
      Par conséquent, la masse totale d'ordures ménagères résiduelles est : $$12700\times 0,256\times 0,9839\approx 27986,3$$ L'objectif fixé par la municipalité est atteint en fin de 2024.

 

Partie C


Dans cette partie, tous les résultats demandés seront arrondis à $10^{-3}$. Des contrôles sont effectués afin de vérifier le tri des déchets.

Protocole d'étude

On choisit au hasard $100$ habitations. Des personnels ont ouvert les poubelles de déchets recyclables de ces habitations afin de déterminer s'ils étaient conformes (absence de matériaux non recyclables, de cartons souillés $\ldots$).

Résultats de l'étude

Parmi ces 100 poubelles de déchets recyclables, 7 ont été jugées non conformes.

  1. Déterminer, à l'aide d'un intervalle de confiance avec un niveau de confiance de 95 %, une estimation de la proportion de poubelles de déchets recyclables qui ne sont pas conformes.
  2. La fréquence est égale à  $\1$. La taille $n$ de l'échantillon considéré est égale à  $\2.$
    Comme  $ n =\2$ ,   $n \times \8  $=\3  et $n\times (1-\8)=\4,$ les conditions d'utilisation d'un intervalle de confiance sont réunies.

    En effet on a bien : $$n \geq 30\;;\; n \times \8 \geq 5 \text{ et } n\times (1-\8) \geq 5$$

    L' intervalle de confiance avec un niveau de confiance de 95% est : \[\9 = \left[\8 - 1,96\sqrt{\dfrac{\8(1 - \8)}{n}}~;~\8 + 1,96\sqrt{\dfrac{\8(1 - \8)}{n}} \right]\]
    La fréquence est $\8=\1$.
    L'intervalle de confiance au niveau de 95% est \[\9 = \left[\1 - 1,96\sqrt{\dfrac{\1 (1 - \1 )}{\2}}~;~\1 + 1,96\sqrt{\dfrac{\1 (1 - \1 )}{\2}} \right]\approx[\5~;~\6]\] 

    Soit avec des valeurs approchées à $10^{-3}$ près des bornes de l'intervalle, l'intervalle de confiance est $IC_{100}=[0,019; 0,12]$.
  3. La proportion de poubelles de déchets recyclables qui ne sont pas conformes est-elle nécessairement comprise dans cet intervalle de confiance ?
  4. Un intervalle de confiance avec un niveau de confiance de 95 % ne contient pas nécessairement la propotion $p$ inconnue. Si on extrait un gand nombre d'échantillons de même taille, 95 % des échantillons donnent un intervalle de confiance contenant $p$.
    La proportion de poubelles de déchets recyclables qui ne sont pas conformes n'appartient pas nécessairement à cet intervalle de confiance.
  • Vues: 12477

Bac STI2D Polynésie 9 juin 2016

  

Exercice 1 3 points


QCM

Cet exercice est un questionnaire à choix multiples. Pour chacune des questions suivantes, une seule des quatre réponses proposées est exacte. Aucune justification n'est demandée. Une bonne réponse rapporte $1$ point. Une mauvaise réponse, plusieurs réponses ou l'absence de réponse à une question ne rapportent ni n'enlèvent de point. Pour répondre, vous recopierez sur votre copie le numéro de la question et la seule réponse choisie.
Dans cet exercice, $\text{i}$ désigne le nombre complexe de module 1 et d'argument $\dfrac{\pi}{2}$.

  1. L'écriture exponentielle du nombre complexe $z = \dfrac{-3\text{i}}{1 + \text{i}}$ est :
    \[ \textbf{a.} z = \frac{3\sqrt{2}}{2}\text{e}^{-\text{i}\frac{5\pi}{4}}\hspace{2cm}\textbf{b.}z = - \frac{3\sqrt{2}}{2}\text{e}^{\text{i}\frac{5\pi}{4}}\hspace{2cm} \textbf{c.} z =\frac{3\sqrt{2}}{2}\text{e}^{\text{i}\frac{5\pi}{4}} \hspace{2cm} \textbf{d.} z = \frac{3\sqrt{2}}{2}\text{e}^{\text{i}\frac{\pi}{4}} \]
  2. Soit $f$ la fonction définie pour tout réel $t$ positif par : $f(t) = 8\text{e}^{-0,12t  }+11$. La valeur moyenne de $f$ arrondie à $10^{-1}$ sur l'intervalle [0;24] est :
    $$ \textbf{a. }15,2\hspace{2cm} \textbf{b. }13,6 \hspace{2cm} \textbf{c }16,7 \hspace{2cm}\textbf{d. } 11,2$$
  3. On donne dans un repère orthonormé les points : A(0;2) ; B(1;3) ; C$(-2;1)$ et D$(-1;0)$. Le produit scalaire $\vec{\text{AB}} \cdot \vec{\text{CD}}$ est égal à :
    $$ \textbf{a.} \vec{\text{AB}} \cdot \vec{\text{CD}} = 0 \hspace{2cm}\hspace{2cm}\hspace{2cm} \textbf{b.} \vec{\text{AB}} \cdot \vec{\text{CD}} = \vec{0} \hspace{2cm}\hspace{2cm} \textbf{c.} \vec{\text{AB}} \cdot \vec{\text{CD}} = - 2 \hspace{2cm} \textbf{d.} \vec{\text{AB}} \cdot \vec{\text{CD}} = \vec{\text{AD}}$$

Exercice 1 3 points


QCM

Cet exercice est un questionnaire à choix multiples. Pour chacune des questions suivantes, une seule des quatre réponses proposées est exacte. Aucune justification n'est demandée. Une bonne réponse rapporte $1$ point. Une mauvaise réponse, plusieurs réponses ou l'absence de réponse à une question ne rapportent ni n'enlèvent de point. Pour répondre, vous recopierez sur votre copie le numéro de la question et la seule réponse choisie.
Dans cet exercice, $\text{i}$ désigne le nombre complexe de module 1 et d'argument $\dfrac{\pi}{2}$.

    1. L'écriture exponentielle du nombre complexe $z = \dfrac{-3\text{i}}{1 + \text{i}}$ est :
      \[ \textbf{a.} z = \frac{3\sqrt{2}}{2}\text{e}^{-\text{i}\frac{5\pi}{4}}\hspace{2cm}\textbf{b.}z = - \frac{3\sqrt{2}}{2}\text{e}^{\text{i}\frac{5\pi}{4}}\hspace{2cm} \textbf{c.} z =\frac{3\sqrt{2}}{2}\text{e}^{\text{i}\frac{5\pi}{4}} \hspace{2cm} \textbf{d.} z = \frac{3\sqrt{2}}{2}\text{e}^{\text{i}\frac{\pi}{4}} \]
    2. Bonne réponse : c

 

      On met tout d'abord $z$ sous forme algégrique $z = \dfrac{-3\text{i}}{1 + \text{i}}$ sous forme algébrique : \begin{align*} z &= \dfrac{-3\text{i}}{1 + \text{i}}\\ &= \dfrac{-3\text{i}(1 - \text{i})}{(1 + \text{i})(1 - \text{i})}\\ &=\dfrac{-3\text{i} +3\text{i}^2 }{(1^2 + 1^2)} \\ &=\dfrac{-3-3\text{i} }{2} \end{align*} $z=a+\text{i}b$ où $a= -\dfrac{3}{2}$ et $b= -\dfrac{3}{2}$
      • Module : \begin{align*} |z| &= \sqrt{\left (\dfrac{3}{2}\right )^2+\left (\dfrac{3}{2}\right )^2}\\ &= \sqrt{ \dfrac{9}{4} +\dfrac{9}{4}}\\ &=\sqrt{ \dfrac{18}{4} } \\ &= \dfrac{3\sqrt{2}}{2} \end{align*}
      • Argument :
        $$\left\lbrace \begin{array}{l} \cos \theta=\dfrac{a}{r}=\dfrac{-\dfrac{3}{2}}{\dfrac{3\sqrt{2}}{2}}= -\dfrac{3}{2}\times \dfrac{2}{3\sqrt 2}=-\dfrac{\sqrt 2}{2}\\ \sin \theta=\dfrac{b}{r}=\dfrac{-\dfrac{3}{2}}{\dfrac{3\sqrt{2}}{2}}= -\dfrac{3}{2}\times \dfrac{2}{3\sqrt 2}=-\dfrac{\sqrt 2}{2} \end{array} \right.$$ Donc $\theta = -\dfrac{3\pi}{4}$ convient ou en ajoutant $2\pi$,$\theta = -\dfrac{3\pi}{4}+2\pi= \dfrac{5\pi}{4}$
      On déduit donc \begin{align*} z &=\left [\dfrac{3\sqrt 2}{2} ;\dfrac{5\pi}{4} \right ] \\ &= \dfrac{3\sqrt 2}{2}\left (\cos\left (\dfrac{5\pi}{4}\right )+\text{i} \sin\left (\dfrac{5\pi}{4}\right )\right ) \\ &= \frac{3\sqrt{2}}{2}\text{e}^{\text{i}\frac{5\pi}{4}} \end{align*}
      • On utilise la calculatrice :( en mode degré) pour les vieilles TI83 ...
      • On utilise la calculatrice :( en mode radian) pour la TI83 Premium ...
    1. Soit $f$ la fonction définie pour tout réel $t$ positif par : $f(t) = 8\text{e}^{-0,12t  }+11$. La valeur moyenne de $f$ arrondie à $10^{-1}$ sur l'intervalle [0;24] est :
      $$ \textbf{a. }15,2\hspace{2cm} \textbf{b. }13,6 \hspace{2cm} \textbf{c }16,7 \hspace{2cm}\textbf{d. } 11,2$$
    2. Bonne réponse : b

 

      Une méthode rapide ? On utilise la calculatrice :
      Le calcul direct : la valeur moyenne d'une fonction $f$ sur $[ a; b]$ est $V_{Moy}= \dfrac{1}{b-a}\displaystyle\int_a^b \; f(t)\; dt$

 

      Ici $f(t)= 8\text{e}^{-0,12t  }+11$, une primitive est donc la fonction $F$ dénie sur $\mathbb R$ par $F(t) = \dfrac{8}{-0,12}\text{e}^{-0,12t  }+11t$ \begin{align*} V_{Moy} &= \dfrac{1}{24}\displaystyle\int_0^{24} \; f(t)\; dt \\ &= \dfrac{1}{24}\left (F(24)-F(0)\right )\\ F(24)&= \dfrac{8}{-0,12}\text{e}^{-0,12\times 24 }+11\times 24\approx 260,26 \\ F(0)&= \dfrac{8}{-0,12}\text{e}^{ 0}+11\times 0\approx -66.6 7 \\ V_{Moy} &\approx \dfrac{1}{24} (326.92)\approx 13,6 \end{align*}
    1. On donne dans un repère orthonormé les points : A(0;2) ; B(1;3) ; C$(-2;1)$ et D$(-1;0)$. Le produit scalaire $\vec{\text{AB}} \cdot \vec{\text{CD}}$ est égal à :
      $$ \textbf{a.} \vec{\text{AB}} \cdot \vec{\text{CD}} = 0 \hspace{2cm}\hspace{2cm}\hspace{2cm} \textbf{b.} \vec{\text{AB}} \cdot \vec{\text{CD}} = \vec{0} \hspace{2cm}\hspace{2cm} \textbf{c.} \vec{\text{AB}} \cdot \vec{\text{CD}} = - 2 \hspace{2cm} \textbf{d.} \vec{\text{AB}} \cdot \vec{\text{CD}} = \vec{\text{AD}}$$
    2. Bonne réponse : a

 

    On a $\vec{AB} \begin{pmatrix} x_B-x_A \\y_B-y_A \end{pmatrix}= \begin{pmatrix} 1-0 \\3-2 \end{pmatrix}= \begin{pmatrix} 1 \\1 \end{pmatrix}$ de même on obtient $\vec{CD} \begin{pmatrix} 1 \\-1 \end{pmatrix}$ \begin{align*} \vec{\text{AB}} \cdot \vec{\text{CD}} &= XX'+YY'\\ &= 1\times 1 + (-1)\times 1\\ &= 1-1 \\ &=0 \end{align*}

Exercice 2 6 points


Suites


L'énergie photovoltaïque voit son coût baisser de façon importante depuis plusieurs années, ce qui engendre une croissance forte de ce secteur. L'évolution de la puissance solaire photovoltaïque installée dans le monde entre fin 2004 et fin 2015 est résumée dans le graphique ci-dessous : 

  1. Calculer les pourcentages d'augmentation annuels entre 2013 et 2014 ainsi qu'entre 2014 et 2015 $\left(\text{arrondir à } 10^{-1}\right)$.
  2. On se propose d'estimer la puissance solaire photovoltaïque installée dans le monde dans les 15 ans à venir, si le taux de croissance annuel reste constant et égal à 30%. On note $P_n$ la puissance solaire photovoltaïque installée dans le monde, en GW, à la fin de l'année $2015 + n$. On a ainsi $P_0 = 233$.
    1. Calculer $P_1$ puis $P_2$ $\left(\text{arrondir à } 10^{-1}\right)$.
    2. Exprimer $P_{n+1}$ en fonction de $P_n$.
    3. En déduire la nature de la suite $\left(P_n\right)$ et donner ses éléments caractéristiques.
    4. Exprimer $P_n$ en fonction de $n$.
    5. Calculer la puissance solaire photovoltaïque, en GW, installée dans le monde fin 2025 (arrondir à l'unité).
    6. Quel est le pourcentage global d'augmentation de cette puissance solaire mondiale entre 2015 et 2025 (arrondir à l'unité) ?
  3. On veut déterminer l'année durant laquelle la puissance solaire photovoltaïque installée dans le monde atteindrait 16000 GW. Pour atteindre cette puissance, les panneaux photovoltaïques occuperaient au sol l'équivalent d'un carré de 400km de côté et suffiraient pour produire toute l'électricité consommée dans le monde (consommation domestique, industrielle et des transports).
    1. On considère l'algorithme ci-dessous. Recopier et compléter les lignes 3 et 7 afin que cet algorithme réponde à la question posée. $$ \begin{array}{|l |l|}\hline 1/& \text{ Affecter à } N \text{ la valeur } 0 \\ 2/& \text{ Affecter à } P \text{ la valeur } 233\\ 3/& \text{ Tant que } \ldots\\ 4/&\text{ Affecter à } N \text{ la valeur } N +1 \\ 5/&\text{ Affecter à } P \text{ la valeur } P \times 1,30 \\ 6/&\text{ Fin Tant que }\\ 7/& \text{ Afficher } \ldots\\ \hline \end{array} $$
    2. En faisant tourner cet algorithme complété, déterminer l'année durant laquelle la puissance solaire photovoltaïque installée dans le monde dépasserait 16000 GW.
    3. Proposer une autre méthode, directe et non algorithmique, pour répondre à la question précédente en détaillant la démarche utilisée.

Exercice 2 6 points


Suites


L'énergie photovoltaïque voit son coût baisser de façon importante depuis plusieurs années, ce qui engendre une croissance forte de ce secteur. L'évolution de la puissance solaire photovoltaïque installée dans le monde entre fin 2004 et fin 2015 est résumée dans le graphique ci-dessous : 

  1. Calculer les pourcentages d'augmentation annuels entre 2013 et 2014 ainsi qu'entre 2014 et 2015 $\left(\text{arrondir à } 10^{-1}\right)$.
  2. $\dfrac{180}{139} \approx 1,295 $ et $\dfrac{233}{180}\approx 1,294$.
    La puissance solaire photovoltaïque a augmenté de 29,5 % entre 2013 et 2014 et de 29,4 % entre 2014 et 2015.

    On se propose d'estimer la puissance solaire photovoltaïque installée dans le monde dans les 15 ans à venir, si le taux de croissance annuel reste constant et égal à 30%. On note $P_n$ la puissance solaire photovoltaïque installée dans le monde, en GW, à la fin de l'année $2015 + n$. On a ainsi $P_0 = 233$.
    1. Calculer $P_1$ puis $P_2$ $\left(\text{arrondir à } 10^{-1}\right)$.
    2. On calcule $P_1= P_0 + 30\%\times P_0 = 233\times \left(1+\dfrac{30}{100}\right)= 1,3 \times 233 = 302,9$
      $P_2= P_1 + 30\%\times P_1 = 302,9\times \left(1+\dfrac{30}{100}\right)= 1,3 \times 302,9 \approx 393,8$
      La puissance solaire photovoltaïque installée dans le monde, sera de 302,9 GW, à la fin de l'année $2016$, et d'environ 393,8 GW, à la fin de l'année $2016$ .
    3. Exprimer $P_{n+1}$ en fonction de $P_n$.
    4. $\begin{align*} P_{n+1}&= P_n + 30\%\times P_n\\ & = \left(1+\dfrac{30}{100}\right)\times P_n\\ &= 1,3P_n \end{align*}$
    5. En déduire la nature de la suite $\left(P_n\right)$ et donner ses éléments caractéristiques.
    6. Ayant pour tout entier naturel $n$ ; $P_{n+1}=1,3P_n$, on déduit que la suite $\left(P_n\right)$ est géométrique de raison 1,3.
    7. Exprimer $P_n$ en fonction de $n$.
    8. Comme la suite $\left(P_n\right)$ est géométrique, on a $P_n=q^n \times P_0$ $$P_n=1,3^n \times 233$$
    9. Calculer la puissance solaire photovoltaïque, en GW, installée dans le monde fin 2025 (arrondir à l'unité).
    10. La puissance solaire photovoltaïque, en GW, installée dans le monde fin 2025 est donnée par $P_{10}=1,3^{10}\times 233\approx 3212$
      La puissance solaire photovoltaïque installée dans le monde, sera d'environ 3212 GW, à la fin de l'année $2025$ .
    11. Quel est le pourcentage global d'augmentation de cette puissance solaire mondiale entre 2015 et 2025 (arrondir à l'unité) ?
    12. On calcule $t$ tel que $P_{10}=(1+t)P_0$ donc $1+t = \dfrac{P_{10}}{P_0}\approx \dfrac{3212}{233}\approx 13,7854$
      On déduit $t\approx 12,79$
      Lle pourcentage global d'augmentation de cette puissance solaire mondiale entre 2015 et 2025 est donc d'environ 1279%.
  3. On veut déterminer l'année durant laquelle la puissance solaire photovoltaïque installée dans le monde atteindrait  16000 GW. Pour atteindre cette puissance, les panneaux photovoltaïques occuperaient au sol l'équivalent d'un carré de 400km de côté et suffiraient pour produire toute l'électricité consommée dans le monde (consommation domestique, industrielle et des transports).
    1. On considère l'algorithme ci-dessous. Recopier et compléter les lignes 3 et 7 afin que cet algorithme réponde à la question posée. $$ \begin{array}{|l |l|}\hline 1/& \text{ Affecter à } N \text{ la valeur } 0 \\ 2/& \text{ Affecter à } P \text{ la valeur } 233\\ 3/& \text{ Tant que } \ldots\\ 4/&\text{ Affecter à } N \text{ la valeur } N +1 \\ 5/&\text{ Affecter à } P \text{ la valeur } P \times 1,30 \\ 6/&\text{ Fin Tant que }\\ 7/& \text{ Afficher } \ldots\\ \hline \end{array} $$
    2. $$ \begin{array}{|l |l|}\hline 1/& \text{ Affecter à } N \text{ la valeur } 0 \\ 2/& \text{ Affecter à } P \text{ la valeur } 233\\ 3/& \text{ Tant que } P< 16\; 000\\ 4/&\text{ Affecter à } N \text{ la valeur } N +1 \\ 5/&\text{ Affecter à } P \text{ la valeur } P \times 1,30 \\ 6/&\text{ Fin Tant que }\\ 7/& \text{ Afficher } 2015+N\\ \hline \end{array} $$
    3. En faisant tourner cet algorithme complété, déterminer l'année durant laquelle la puissance solaire photovoltaïque installée dans le monde dépasserait 16000 GW.
    4. On utilise la calculatrice :
      Conclusion : La puissance solaire photovoltaïque installée dans le monde dépasserait 16000 GW pour la première fois en 2032.
    5. Proposer une autre méthode, directe et non algorithmique, pour répondre à la question précédente en détaillant la démarche utilisée.
    6. $$\begin{array}{ll} P_n> 16 000 &\iff 1,3^n \times 233 > 16 000\\ &\iff 1,3^n > \dfrac{16 000}{233}\\ &\iff n \ln(1,3) > \ln \left(\dfrac{16 000}{233}\right)\\ &\iff n > \dfrac{ \ln \left(\dfrac{16 000}{233}\right)}{ \ln (1,3)} \\ \end{array}$$ REMARQUE : Comme $\dfrac{ \ln \left(\dfrac{16 000}{233}\right)}{ \ln (1,3)} \approx 16,12$, le plus petit entier $N$ tel que $N>\dfrac{ \ln \left(\dfrac{16 000}{233}\right)}{ \ln (1,3)} $ est $N=17$. On retrouve le résultat de la question précédente.

 


Exercice 3 5 points


Probabilités


Deux amis ont monté un atelier associatif pour réparer des vélos. Le but de cette association est que chaque adhérent puisse venir réparer son vélo dans cet atelier avec l'aide d'un spécialiste. Le matériel et les outils sont fournis. Les trois parties de cet exercice peuvent être traitées de manière indépendante

Partie A : les roulements à billes


Nos deux amis commandent régulièrement des lots de $60$ roulements à billes pour les vélos. Ils ont constaté que, lors de leur dernière livraison, sur le lot des $60$ roulements à billes, $3$ étaient défectueux. Ils s'inquiètent donc de la fiabilité du fabricant. Le contrat précise que seulement 4% des pièces sont défectueuses.

  1. Calculer la fréquence des pièces défectueuses dans le dernier lot. On considère que les pièces constituant ce lot forment un échantillon prélevé de façon aléatoire dans un stock dans lequel 4% des pièces sont défectueuses.
  2. Déterminer l'intervalle de fluctuation asymptotique à 95% de la fréquence des roulements à billes non conformes dans un échantillon de $60$ roulements. Les valeurs approchées seront arrondies à $10^{-2}$.
    On rappelle que l'intervalle de fluctuation asymptotique à 95% sur un échantillon de taille $n$, avec $p$ la proportion de pièces défectueuses sur la population, est : \[\left[p - 1,96\sqrt{\dfrac{p(1 - p)}{p}}~;~p + 1,96\sqrt{\dfrac{p(1 - p)}{p}}\right]\]
  3. Nos amis ont-ils raison de s'inquiéter ? Justifier votre réponse.
Partie B : les billes


Nos amis se demandent s'ils ne devraient pas plutôt commander des billes pour réparer les roulements évoqués dans la partie A. Ils commandent une grande quantité de billes de 6~mm de diamètre. Malheureusement, certaines présentent un défaut de diamètre. Ils s'aperçoivent qu'ils ne peuvent utiliser que les billes mesurant entre $5,9$ mm et $6,1$ mm. Sur la note du fabricant est indiqué que la variable aléatoire $D$ qui, à chaque bille, lui associe son diamètre, suit la loi normale d'espérance $\mu = 6$ mm et d'écart-type $\sigma = 0,05$ mm.
Question : Calculer la probabilité $P(5,9 \leqslant D \leqslant 6,1)$. Le résultat sera arrondi à $10^{-2}$.

Partie C : les chaînes de vélo


Un tableau est mis à disposition pour permettre aux utilisateurs de savoir quand ils doivent changer leur chaîne de vélo. Par exemple, pour une personne utilisant son vélo en ville $\left(\text{vitesse moyenne } 16~ \text{km.h}^{-1}\right)$ environ 2 heures par jour, la durée de vie moyenne de la chaîne est de 625~jours. On admet que la durée de vie en jour, d'une chaîne de vélo pour un tel utilisateur est une variable aléatoire $X$ qui suit une loi exponentielle de paramètre $\lambda$. On rappelle que la probabilité que $X$ soit inférieure ou égale à $t$ (exprimé en jour) vaut : $P(X \leqslant t) = 1 - \text{e}^{- \lambda t}$.

  1. Démontrer que $\lambda = 0,0016 $.
  2. Le graphique en ANNEXE 1 représente la fonction de densité de la loi exponentielle de paramètre $\lambda = 0,0016 $ $\left(\text{exprimé en jour}^{-1}\right)$.
    1. Représenter sur ce graphique la probabilité que $X$ soit comprise entre 350 jours et 700 jours.
    2. Calculer la probabilité que $X$ soit comprise entre $350$ jours et $700$ jours. Arrondir le résultat à $10^{-3}$.
  3. Calculer la probabilité que $X$ soit de moins de $550$ jours. Arrondir à $10^{-3}$.
  4. Déterminer la valeur de $x$ pour que $P(X < x) = 0,8$. Le résultat sera arrondi à l' unité. Interpréter ce résultat en le resituant dans le contexte.

Annexe à rendre avec la copie

 Ex3


Correction de l'exercice 3 (5 points)


Probabilités


Deux amis ont monté un atelier associatif pour réparer des vélos. Le but de cette association est que chaque adhérent puisse venir réparer son vélo dans cet atelier avec l'aide d'un spécialiste. Le matériel et les outils sont fournis. Les trois parties de cet exercice peuvent être traitées de manière indépendante

Partie A : les roulements à billes


Nos deux amis commandent régulièrement des lots de $60$ roulements à billes pour les vélos. Ils ont constaté que, lors de leur dernière livraison, sur le lot des $60$ roulements à billes, $3$ étaient défectueux. Ils s'inquiètent donc de la fiabilité du fabricant. Le contrat précise que seulement 4% des pièces sont défectueuses.

    1. Calculer la fréquence des pièces défectueuses dans le dernier lot. On considère que les pièces constituant ce lot forment un échantillon prélevé de façon aléatoire dans un stock dans lequel 4% des pièces sont défectueuses.
    2. La fréquence des pièces défectueuses est $f= \dfrac{3}{60}= \dfrac{1}{20}=0,05$
    3. Déterminer l'intervalle de fluctuation asymptotique à 95% de la fréquence des roulements à billes non conformes dans un échantillon de $60$ roulements. Les valeurs approchées seront arrondies à $10^{-2}$.
      On rappelle que l'intervalle de fluctuation asymptotique à 95% sur un échantillon de taille $n$, avec $p$ la proportion de pièces défectueuses sur la population, est : \[\left[p - 1,96\sqrt{\dfrac{p(1 - p)}{p}}~;~p + 1,96\sqrt{\dfrac{p(1 - p)}{p}}\right]\]
    4. La proportion $p$ est égale à $\1$. La taille $n$ de l'échantillon considéré est égale à $\2.$
      Comme $ n =\2$ , $n \times p $=\3 et $n\times (1-p)=\4,$ les conditions d'utilisation d'un intervalle de fluctuation asymptotique ne sont pas sont réunies !

      En effet on a bien : $$n \geq 30\;;\; n\times (1-p) \geq 5 \text{ mais } n \times p < 5$$


      L'intervalle de fluctuation asymptotique au seuil de $95\% $ est : $$I_{\2} = \left[\1 - 1,96\sqrt{\dfrac{\1\times \5}{\2}}~;~\1 + 1,96\sqrt{\dfrac{\1\times \5}{\2}} \right]$$

       

       

      On obtient $I_{60}\approx [ 0; 0,0896]$
    5. Nos amis ont-ils raison de s'inquiéter ? Justifier votre réponse.
    6. Comme $f\in I_{60}$ . Nos amis n'ont pas raison de s'inquiéter. L'écart constaté entre le 5% observé dans l'échantillon et e 4% attendu est dû à des fluctuations d'échantillonnage.
    Partie B : les billes


    Nos amis se demandent s'ils ne devraient pas plutôt commander des billes pour réparer les roulements évoqués dans la partie A. Ils commandent une grande quantité de billes de 6 mm de diamètre. Malheureusement, certaines présentent un défaut de diamètre. Ils s'aperçoivent qu'ils ne peuvent utiliser que les billes mesurant entre $5,9$ mm et $6,1$ mm. Sur la note du fabricant est indiqué que la variable aléatoire $D$ qui, à chaque bille, lui associe son diamètre, suit la loi normale d'espérance $\mu = 6$ mm et d'écart-type $\sigma = 0,05$ mm.

    Question : Calculer la probabilité $P(5,9 \leqslant D \leqslant 6,1)$. Le résultat sera arrondi à $10^{-2}$.
    • Méthode 1 :

      2ND DISTR 2NORMALFRép( \1 , \2,\3,\4)EXE
      Avec une calculatrice de type TI

      $$NormalFR\text{é}p(\1,\2,\3,\4) \approx \5$$

      $$P(\1 \leq \6 \leq \2)\approx \5 \text{ à } 10^{-\7} \text{ près.}$$

       

    • Méthode 2 :
      On remarque que $P(5,9 \leqslant D \leqslant 6,1)= P(\mu-2\sigma \leqslant D \leqslant \mu+2\sigma)\approx 0,954$ d'après un résultat du cours.
    Partie C : les chaînes de vélo


    Un tableau est mis à disposition pour permettre aux utilisateurs de savoir quand ils doivent changer leur chaîne de vélo. Par exemple, pour une personne utilisant son vélo en ville $\left(\text{vitesse moyenne } 16~ \text{km.h}^{-1}\right)$ environ 2 heures par jour, la durée de vie moyenne de la chaîne est de 625 jours. On admet que la durée de vie en jour, d'une chaîne de vélo pour un tel utilisateur est une variable aléatoire $X$ qui suit une loi exponentielle de paramètre $\lambda$. On rappelle que la probabilité que $X$ soit inférieure ou égale à $t$ (exprimé en jour) vaut : $P(X \leqslant t) = 1 - \text{e}^{- \lambda t}$.

    1. Démontrer que $\lambda = 0,0016 $.
      • Méthode 1 :
        On sait que $E(X)= \dfrac{1}{\lambda}$ donc $\lambda = \dfrac{1}{E(X)} =\dfrac{1}{625}=0,0016$
      • Méthode 2 :
        La densité d'une loi exponentielle de paramètre $\lambda$ est définie sur $[0; +\infty[$ par $f(x)= \lambda\text{e}^{- \lambda x}$
        On a donc $f(0)=\lambda \text{e}^{0}= \lambda$
        Par ailleurs on lit graphiquement $f(0)= 0,0016$ , on déduit donc $$\lambda = 0,0016 $$
    2. Le graphique en ANNEXE 1 représente la fonction de densité de la loi exponentielle de paramètre $\lambda = 0,0016 $ $\left(\text{exprimé en jour}^{-1}\right)$.
      1. Représenter sur ce graphique la probabilité que $X$ soit comprise entre 350 jours et 700 jours.
      2. Calculer la probabilité que $X$ soit comprise entre $350$ jours et $700$ jours. Arrondir le résultat à $10^{-3}$.
      3. \begin{align*} P(350\leq X\leq 700) &=\displaystyle\int_{350}^{700} f(x)\; dx \\ &= \displaystyle\int_{350}^{700} \lambda \text{e}^{-\lambda x}\; dx \\ &=\left [ -\text{e}^{-\lambda x}\right ]_{350}^{700}\\ &= -\text{e}^{-700 \lambda }-\left (-\text{e}^{-350 \lambda }\right )\\ &=\text{e}^{-350 \times 0,0016 }-\text{e}^{-700 \times 0,0016 }\\ &\approx 0,245\\ \end{align*}
    3. Calculer la probabilité que $X$ soit de moins de $550$ jours. Arrondir à $10^{-3}$.
    4. \begin{align*} P( X\leq 550) &=\displaystyle\int_{0}^{550} f(x)\; dx \\ &= \displaystyle\int_{0}^{550} \lambda \text{e}^{-\lambda x}\; dx \\ &=\left [ -\text{e}^{-\lambda x}\right ]_{0}^{550}\\ &= -\text{e}^{-550 \lambda }-\left (-\text{e}^{0 }\right )\\ &=1-\text{e}^{-550 \times 0,0016 }\\ &\approx 0,5855\\ \end{align*}
    5. Déterminer la valeur de $x$ pour que $P(X < x) = 0,8$. Le résultat sera arrondi à l' unité. Interpréter ce résultat en le resituant dans le contexte .
    6. On cherche $x$ tel que $P(X < x) = 0,8$
      On sait que $P(X < x) = 1-\text{e}^{-\lambda x}$ \begin{align*} P( X\leq x) =0,8&\iff 1-\text{e}^{-\lambda x} =0,8 \\ &\iff \text{e}^{-\lambda x} =0,8 \\ &\iff -\lambda x =\ln(0,2) \\ &\iff x=-\dfrac{\ln(0,2)}{\lambda}\\ &\iff x=-\dfrac{\ln(0,2)}{0,0016}\\ &x\approx 1006\\ \end{align*} Il y a 80% de chance que la durée de vie moyenne de la chaîne soit inférieure à 1006 jours.

    Exercice 4 6 points


    Fonctions

     

    Partie A : Lecture graphique


    On considère la courbe $C$ associée à une fonction $f$ représentée en ANNEXE 2 avec la droite T, tangente à la courbe $C$ au point d'abscisse $0$.

    1. Résoudre graphiquement sur l'intervalle $[- 1~;~1,5]$ et avec la précision permise par le dessin les deux inéquations suivantes:
      1. $f(x) \geqslant 1$
      2. $f'(x) \geqslant 0$.
      1. Donner l'équation. de la tangente T à la courbe $C$ au point de coordonnées (0 ; 1) en sachant que cette tangente passe par le point de coordonnées (2 ; 7).
      2. En déduire le nombre dérivé $f'(0)$.

     

    Partie B : Étude de la fonction $f$


    Soit $f$ la fonction définie sur $\mathbb R$ par la relation $f(x) = \text{e}^{-2x} + 5x$.

    1. Déterminer, en la justifiant, la limite de $f$ en $+ \infty$. On admet pour la suite que la limite de $f$ en $- \infty$ est $+ \infty$.
    2. Calculer $f'(x)$ et étudier son signe sur $\mathbb R$.
    3. En déduire le tableau des variations de la fonction $f$ sur $\mathbb R$.
      1. Déterminer à partir du tableau des variations le nombre de solutions de l'équation $f(x) = 2$.
      2. Donner une valeur arrondie à $10^{-2}$ près de chaque solution.

     

    Partie C. : Calcul d'aire


    On admet :

    • que la courbe $C$ de la partie A est la représentation de la fonction $f$ définie dans la partie B ;
    • que la courbe $C$ se situe « au-dessus » de la droite tangente T sur $\mathbb R$.

    L'objectif de cette partie est de déterminer par un calcul l'aire $\mathcal{A}$ comprise entre la courbe $C$, la droite T et les droites verticales d'équations $x = 0$ et $x = 1,5$.

    1. Hachurer sur le dessin, en ANNEXE 2, l'aire $\mathcal{A}$ que l'on veut déterminer.
      1. Déterminer une primitive de la fonction $g$ définie sur $\mathbb R$ par : \[\text{pour tout réel }\:x,\: g(x) = \text{e}^{-2x} + 2x - 1.\]
      2. Justifier que l'aire $\mathcal{A}$ recherchée vaut, en unité d'aire: \[\mathcal{A} = \displaystyle\int_0^{1,5} g(x)\:\text{d}x.\]
      3. En déduire la valeur exacte puis l'arrondi à $10^{- 2}$ de $\mathcal{A}$.

     

    Exercice 4 Parties A et C

     


    Exercice 4 6 points


    Fonctions

     

    Partie A : Lecture graphique


    On considère la courbe $C$ associée à une fonction $f$ représentée en ANNEXE 2 avec la droite T, tangente à la courbe $C$ au point d'abscisse $0$.

    1. Résoudre graphiquement sur l'intervalle $[- 1~;~1,5]$ et avec la précision permise par le dessin les deux inéquations suivantes:
      1. $f(x) \geqslant 1$
      2. Les solutions de l'inéquation $f(x)\geq 1$ sont les abscisses des points situés au dessus de la droite d'équation $y=1$. On lit àla précision du dessin : $$\mathcal{S}= [-1; -0,8] \cup [0; +\infty[$$
      3. $f'(x) \geqslant 0$.
      4. D'après le graphique $f$ est croissante sur $[x(S); +1,5]$.
        L'ensemble des solutions de l'inéquation $f'(x)\geq 0$ est : $$\mathcal{S}= [-0,4; 1,5]$$
      1. Donner l'équation. de la tangente T à la courbe $C$ au point de coordonnées (0 ; 1) en sachant que cette tangente passe par le point de coordonnées P(2 ; 7).
      2. $T$ a une équation du type $y=mx+p$ où \begin{align*} m&=\dfrac{y_P-y_E}{x_P-x_E} \\ &= =\dfrac{4-1}{1-0} \\ &=3\\ \end{align*} Donc $T: y=3x+p$ Comme $E(1,0)\in T$ on déduit $y_E=3x_E+p$ ce qui donne $1=3\times 0 + 1$ soit $p=1$
      3. En déduire le nombre dérivé $f'(0)$.
      4. $f'(0)$ est le coefficient directeur de la tangente $T$ au point d'abscisse 0
        d'après la question précédente $f'(0)= 3$.

     

    Partie B : Étude de la fonction $f$


    Soit $f$ la fonction définie sur $\mathbb R$ par la relation $f(x) = \text{e}^{-2x} + 5x$.

    1. Déterminer, en la justifiant, la limite de $f$ en $+ \infty$. On admet pour la suite que la limite de $f$ en $- \infty$ est $+ \infty$.
    2. $\left.\begin{array}{l} \lim\limits_{-2x \to +\infty} =-\infty\\ \lim\limits_{t \to -\infty}~\text{e}^t=0 \end{array}\right\}$ par composée on obtient: $\lim\limits_{x \to +\infty}\text{e}^{-2x} =0$
      $\left.\begin{array}{l} \lim\limits_{x \to +\infty} \text{e}^{-2x} =0\\ \lim\limits_{ x \to +\infty}~5x=+\infty \end{array}\right\}$ par somme on obtient: $\lim\limits_{x \to +\infty} f(x)=+\infty$
    3. Calculer $f'(x)$ et étudier son signe sur $\mathbb R$.
    4. Comme $\left(\text{e}^u\right)'=u'\text{e}^u$, on obtient $$f'(x)=-2\text{e}^{-2x}+5$$ Etudions le signe de la dérivée :
      • \begin{align*} f'(x)=0 &\iff -2\text{e}^{-2x}+5=0 \\ &\iff - 2\text{e}^{-2x} =-5 \\ &\iff \text{e}^{-2x} = \dfrac{5}{2}\\ &\iff -2x =\ln\left (\dfrac{5}{2}\right )\\ &\iff x=-\dfrac{1}{2}\ln\left (\dfrac{5}{2}\right )\\ \end{align*}
      • \begin{align*} f'(x)>0 &\iff -2\text{e}^{-2x}+5>0 &\\ &\iff - 2\text{e}^{-2x} >-5 &\\ &\iff \text{e}^{-2x} < \dfrac{5}{2}& \text{en divisant par } -2< 0\\ &\iff -2x < \ln\left (\dfrac{5}{2}\right )&\text{en appliquant } \ln \text{strictement croissante sur }]0; +\infty[\\ &\iff x>-\dfrac{1}{2}\ln\left (\dfrac{5}{2}\right )&\text{en divisant par } -2< 0 \\ \end{align*}
    5. En déduire le tableau des variations de la fonction $f$ sur $\mathbb R$.
    6. On déduit le tableau de variations de $f$ sur $\mathbb R$:

      $f\left(-\dfrac{1}{2}\ln\left (\dfrac{5}{2}\right )\right ) =f(\alpha)=\text{e}^{-2\alpha} + 5\alpha $
      Comme $f'(\alpha)=0$ on déduit $-2\text{e}^{-2\alpha} + 5=0$ donc $ \text{e}^{-2\alpha}=\dfrac{5}{2}$
      Ainsi $f(\alpha)=\dfrac{5}{2} - \dfrac{5}{2}\ln\left (\dfrac{5}{2}\right ) $
      1. Déterminer à partir du tableau des variations le nombre de solutions de l'équation $f(x) = 2$.
      2. D'après le tableau de variation de $f$ l'équation $f(x)=2$ a deux solutions :
        • une notée $x_1$ dans $]-\infty; \alpha]$
        • une autre notée $x_2$ dans $[ \alpha; +\infty[$
      3. Donner une valeur arrondie à $10^{-2}$ près de chaque solution.
      4. Avec les outils graphiques de la calculatrice on obtient :
        • $x_1\approx 0,96 $
        • $x_2\approx 0,29$

     

    Partie C. : Calcul d'aire


    On admet :

    • que la courbe $C$ de la partie A est la représentation de la fonction $f$ définie dans la partie B ;
    • que la courbe $C$ se situe « au-dessus » de la droite tangente T sur $\mathbb R$.

    L'objectif de cette partie est de déterminer par un calcul l'aire $\mathcal{A}$ comprise entre la courbe $C$, la droite T et les droites verticales d'équations $x = 0$ et $x = 1,5$.

    1. Hachurer sur le dessin, en ANNEXE 2, l'aire $\mathcal{A}$ que l'on veut déterminer.
      1. Déterminer une primitive de la fonction $g$ définie sur $\mathbb R$ par : \[\text{pour tout réel }\:x,\: g(x) = \text{e}^{-2x} + 2x - 1.\]
      2. On utilise deux résultats du cours :
        • La primitive d'une somme est la somme des primitives.
        • $x\mapsto \text{e}^{ax}$ admat pour primitive $x\mapsto \dfrac{1}{a}\text{e}^{ax}$
        Donc une primitive $G$ de $g$ est définie sur $\mathbb R$ par : \[\text{pour tout réel }\:x,\: G(x) =-\dfrac{1}{2} \text{e}^{-2x} + x^2 - x.\]
      3. Justifier que l'aire $\mathcal{A}$ recherchée vaut, en unité d'aire: \[\mathcal{A} = \displaystyle\int_0^{1,5} g(x)\:\text{d}x.\]
      4. Déterminons tout d'aboerd une équation de T:
        $T: y=f'(0)(x-0)+f(0)$, ici $f(0)=1$ et $f'(0)=3$
        On déduit donc : $$T:y=3x+1$$ D'après l'énoncé la courbe $C$ se situe « au-dessus » de la droite tangente T sur $\mathbb R$,
        L'aire $\mathcal{A}$ est donc
        $$\begin{align*} \mathcal{A} &= \displaystyle\int_0^{1,5} \left(f(x)-y_T\right)\:\text{d}x\\ &= \displaystyle\int_0^{1,5} \left(f(x)-3x-1\right)\:\text{d}x\\ &= \displaystyle\int_0^{1,5} \left(\text{e}^{-2x} + 5x-3x-1\right)\:\text{d}x\\ &= \displaystyle\int_0^{1,5} \left(\text{e}^{-2x} + 2x -1\right)\:\text{d}x\\ &= \displaystyle\int_0^{1,5} g(x)\:\text{d}x. \end{align*}$$
      5. En déduire la valeur exacte puis l'arrondi à $10^{- 2}$ de $\mathcal{A}$.

      6. $\begin{align*} \mathcal{A} &= \displaystyle\int_0^{1,5} g(x)\:\text{d}x.\\ &=G(1,5)-G(0)\\ G(1,5)=-\dfrac{1}{2} \text{e}^{-2\times 1,5} + 1,5^2 - 1,5=-\dfrac{1}{2} \text{e}^{-3}+0,75 &\quad G(0)=-\dfrac{1}{2} \text{e}^{-2\times 0} + 0^2 - 0=-0,5\\ \mathcal{A} &= -\dfrac{1}{2} \text{e}^{-3}+0,75+0,5\\ \mathcal{A} &= -\dfrac{1}{2} \text{e}^{-3}+\dfrac{5}{4}\\ &\approx 1,23\end{align*}$

     

     

  • Vues: 10927

Bac STI2D Métropole 16 juin 2016

Exercice 1 4 points


QCM

Cet exercice est un questionnaire à choix multiples. Pour chacune des questions suivantes, une seule des quatre réponses proposées est exacte. Aucune justification n'est demandée. Une bonne réponse rapporte un point. Une mauvaise réponse, plusieurs réponses ou l'absence de réponse à une question ne rapportent ni n'enlèvent de point.
Indiquer sur la copie le numéro de la question et la lettre correspondant à la réponse.
Le plan complexe est rapporté à un repère orthonormé direct $\left(\text{O},~\vec{u},~\vec{v}\right)$. On note $\text{i}$ le nombre complexe vérifiant $\text{i}^2 =-1$

  1. Un argument du nombre complexe $2 + 2\text{i}$ est égal à :
    1. $-\dfrac{\pi}{4}$
    2. $-\dfrac{9\pi}{4}$
    3. $2\sqrt 2$
    4. $\dfrac{\pi}{4}$
  2. Le nombre complexe $\text{e}^{\text{i}\frac{\pi}{5}}\times \text{e}^{\text{i}\frac{2\pi}{15}}$ est égal à :
    1. $\frac{1}{2}+ \frac{\sqrt 3}{2}\text{i}$
    2. $\frac{\sqrt 3}{2}+\frac{1}{2} \text{i}$
    3. $0,5+0,866 \text{i}$
    4. $0,5+0,8660254038 \text{i}$
  3. On considère les points $A$ et $B$ d'affixes respectives $z_A = 2 \text{e}^{\text{i}\frac{\pi}{3}}$ et $z_B = \frac{5}{2} \text{e}^{\text{i}\frac{5\pi}{6}}$. Le triangle $OAB$ est :
    1. isocèle en $O$
    2. rectangle en $O$
    3. rectangle et isocèle en $B$
    4. isocèle en $B$
  4. Pour tout nombre réel $\theta$, le nombre complexe $\text{e}^{\text{i}\theta} +\dfrac{1}{\text{e}^{\text{i}\theta}} $ est égal à :
    1. $2\cos\left(\theta\right)$
    2. $\cos\left(\theta\right)+\text{i}\sin\left(\theta\right) $
    3. $1$
    4. $2\text{i}\sin\left(\theta\right)$

 


Correction de l'exercice 1 (4 points)


QCM

Cet exercice est un questionnaire à choix multiples. Pour chacune des questions suivantes, une seule des quatre réponses proposées est exacte. Aucune justification n'est demandée. Une bonne réponse rapporte un point. Une mauvaise réponse, plusieurs réponses ou l'absence de réponse à une question ne rapportent ni n'enlèvent de point.
Indiquer sur la copie le numéro de la question et la lettre correspondant à la réponse.
Le plan complexe est rapporté à un repère orthonormé direct $\left(\text{O},~\vec{u},~\vec{v}\right)$ On note $\text{i}$ le nombre complexe vérifiant $\text{i}^2 =-1$

  1. Un argument du nombre complexe $2 + 2\text{i}$ est égal à :
    1. $-\dfrac{\pi}{4}$
    2. $-\dfrac{9\pi}{4}$
    3. $2\sqrt 2$
    4. $\dfrac{\pi}{4}$
  2. $2 + 2\text{i}$
    • Module : \begin{align*} |z| & = \sqrt{ 2^2+2^2}\\ & = \sqrt{8}\\ & =2\sqrt{ 2 } \end{align*}
    • Argument :
      $$\left\lbrace \begin{array}{l} \cos \theta=\dfrac{a}{r}=\dfrac{2}{2\sqrt{ 2 }}= \dfrac{\sqrt 2 }{2} \\ \sin \theta=\dfrac{b}{r}=\dfrac{2}{2\sqrt{ 2 }}= \dfrac{\sqrt 2 }{2}\end{array} \right.$$ Donc $\theta = \dfrac{ \pi}{4}$
    Un argument du nombre complexe $2 + 2\text{i}$ est égal à $\theta = \dfrac{ \pi}{4}$
    • On utilise la calculatrice :( en mode degré) pour les vieilles TI83 ...
      Capturer1 Capturer2
    • Le nombre complexe $\text{e}^{\text{i}\frac{\pi}{5}}\times \text{e}^{\text{i}\frac{2\pi}{15}}$ est égal à :
      1. $\frac{1}{2}+ \frac{\sqrt 3}{2}\text{i}$
      2. $\frac{\sqrt 3}{2}+\frac{1}{2} \text{i}$
      3. $0,5+0,866 \text{i}$
      4. $0,5+0,8660254038 \text{i}$
    • \begin{align*} \text{e}^{\text{i}\frac{\pi}{5}}\times \text{e}^{\text{i}\frac{2\pi}{15}}&=\text{e}^{\text{i}\left (\frac{\pi}{5}+\frac{2\pi}{15}\right )}\\ & =\text{e}^{\text{i}\left (\frac{3\pi}{15}+\frac{2\pi}{15}\right )}\\ &=\text{e}^{\text{i}\frac{\pi}{3}}\\ &=\cos\left (\frac{\pi}{3}\right )+\text{i}\sin\left (\frac{\pi}{3}\right )\\ &=\frac{1}{2}+ \frac{\sqrt 3}{2}\text{i} \end{align*}
    • On considère les points $A$ et $B$ d'affixes respectives $z_A = 2 \text{e}^{\text{i}\frac{\pi}{3}}$ et $z_B = \frac{5}{2} \text{e}^{\text{i}\frac{5\pi}{6}}$. Le triangle $OAB$ est :
      1. isocèle en $O$
      2. rectangle en $O$
      3. rectangle et isocèle en $B$
      4. isocèle en $B$
    • Plusieurs méthodes sont possibles !  On commence par faire une figure :

      \begin{align*} z_A &= 2 \text{e}^{\text{i}\frac{\pi}{3}}\\ &=2\left (\cos\left (\frac{\pi}{3}\right )+\text{i}\sin\left (\frac{\pi}{3}\right )\right )\\ &=2\left (\frac{1}{2}+ \frac{\sqrt 3}{2}\text{i}\right )\\ &=1+\text{i}\sqrt 3\\ \text{Donc }& A(1, \sqrt 3) \end{align*} \begin{align*} z_B& =\frac{5}{2} \text{e}^{\text{i}\frac{5\pi}{6}}\\ &=\frac{5}{2}\left (\cos\left (\frac{5\pi}{6}\right )+\text{i}\sin\left (\frac{5\pi}{6}\right )\right )\\ &=\frac{5}{2}\left (\frac{\sqrt 3}{2}+ \text{i}\frac{1}{2}\right )\\ &=-\dfrac{5\sqrt 3}{4} +\text{i} \dfrac{5}{4}\\ \text{Donc }& B\left (-\dfrac{5\sqrt 3}{4}, \dfrac{5}{4}\right ) \end{align*} On a donc \begin{align*} z_{\vec{0A}} &=z_A-z_O\\ &=z_A\\ &=1+\text{i}\sqrt 3\\ \text{Donc }&\vec{0A} (1, \sqrt 3) \end{align*} \begin{align*} z_{\vec{0B}} &=z_B-z_O\\ &=z_B\\ &=-\dfrac{5\sqrt 3}{4} +\text{i} \dfrac{5}{4}\\ \text{Donc }& \vec{0B}\left (-\dfrac{5\sqrt 3}{4}, \dfrac{5}{4}\right ) \end{align*} Ainsi \begin{align*} \vec{0A}\cdot \vec{0B}& = XX' + YY'\\ &=1\times \left (-\dfrac{5\sqrt 3}{4}\right )+\sqrt 3 \times \dfrac{5}{4}\\ &=-\dfrac{5\sqrt 3}{4}+ \dfrac{5\sqrt 3}{4}\\ &=0 \end{align*} Ayant $\vec{0A}\cdot \vec{0B}=0$ les vecteurs $\vec{0A}$ et $ \vec{0B}$ sont orthogonaux, donc le triangle OAB est rectangle en $O$.
    • Pour tout nombre réel $\theta$, le nombre complexe $\text{e}^{\text{i}\theta} +\dfrac{1}{\text{e}^{\text{i}\theta}} $ est égal à :
      1. $2\cos\left(\theta\right)$
      2. $\cos\left(\theta\right)+\text{i}\sin\left(\theta\right) $
      3. $1$
      4. $2\text{i}\sin\left(\theta\right)$
    • \begin{align*} \text{e}^{\text{i}\theta} +\dfrac{1}{\text{e}^{\text{i}\theta}}&=\text{e}^{\text{i}\theta} +\text{e}^{-\text{i}\theta}& \text{ Prop.}\\ &=2\cos\left(\theta\right)&\text{ Première formule d'Euler}\\ \end{align*}

 


Exercice 2 6 points


Suites

Un centre de vacances possède une piscine de $600\quad m^3$ soit $600\quad 000$ litres. L'eau du bassin contient du chlore qui joue le rôle de désinfectant. Toutefois le chlore se dégrade et 25% de celui-ci disparaît chaque jour, en particulier sous l'effet des ultra-violets et de l'évaporation. Le 31 mai à 9 h, le responsable analyse l'eau du bassin à l'aide d'un kit distribué par un magasin spécialisé.
Le taux de chlore disponible dans l'eau est alors de 1,25 mg/L (milligrammes par litre).

Document
Réglementation des piscines publiques

$$\begin{array}{|c|c|c|} \hline \text{Paramètres contrôlés} & \text{Seuils de qualité réglementaire} &\text{Incidences sur la qualité de l'eau} \\ \hline & \text{Au minimum 2 mg/L} & < \text{2 mg/L : sous chloration}\\ &&\text{Risque de prolifération}\\ && \text{bactérienne dans l eau }\\ \hline \text{Présence de Chlore }& \text{ Au maximum 4 mg/L }&\text{ > 4 mg/L : surchloration }\\ &&\text{Irritation de la peau}\\ \hline \end{array} $$

Source : Agence Régionale de Santé  

A partir du 1$^{\text{er}}$ juin pour compenser la perte en chlore, la personne responsable de l'entretien ajoute, chaque matin à 9 h, 570 g de chlore dans la piscine.
Pour le bien-être et la sécurité des usagers, le responsable souhaite savoir si cet apport journalier en chlore permettra de maintenir une eau qui respecte la réglementation donnée par l' Agence Régionale de Santé pour les piscines publiques.

Partie A
  1. Pour tout entier naturel $n$ on note $u_n$ la quantité de chlore disponible, exprimée en grammes, présente dans l'eau du bassin le $n^{\text{ième}}$ jour suivant le jour de l'analyse, immédiatement après l'ajout de chlore. Ainsi $u_0$ est la quantité de chlore le 31 mai à 9 h et $u_1$ est la quantité de chlore le 1$^{\text{er}}$ juin à 9 h après l'ajout de chlore.
    1. Montrer que la quantité de chlore, en grammes, présente dans l'eau du bassin le 31 mai à 9h est $u_0 = 750$.
      Au regard des recommandations de l'agence régionale de santé, le responsable pouvait-il donner l'accès à la piscine le 31 mai?
    2. Montrer que $u_1 = 1132,5$.
    3. Justifier que pour tout entier naturel $n, u_{n+1} = 0, 75u_n+ 570$
    4. La suite $(u_n)$ est-elle géométrique ?
  2. Soit l'algorithme ci-dessous : $$\begin{array}{|l |l |}\hline \text{Variables } & \\ &u : \text{un nombre réel }\\ &N : \text{un nombre entier naturel }\\ &k : \text{un nombre entier naturel }\\ \text{ Initialisation :}&\\ & \text{Saisir la valeur de } N \\ \text{ Initialisation :}&\\ & u \text{ prend la valeur } 750 \\ \text{ Traitement :}&\\ &\text{ Pour } k \text{ allant de } 1 \text{ à } N\\ &\hspace{0,5cm} u\text{ prend la valeur } 0,75 u +570\\ &\text{ Fin du Pour }\\ \text{ Sortie : }& \text{ Afficher } u \\ \hline \end{array}$$
    1. Quel est le rôle de cet algorithme ?
    2. Recopier et compléter le tableau suivant, par des valeurs exactes, en exécutant cet algorithme \og pas à pas »pour $N=3$. $$\begin{array}{|c|c|c|c|c|} \hline \text{Variables } & \text{ Initialisation }& \text{ Etape 1 } & \text{ Etape 2 } & \text{ Etape 3 } \\ \hline u & 750 &1132,5& & \\ \hline \end{array} $$ Au regard des recommandations de l'agence régionale de santé, au bout de combien de jours la piscine peut-elle être ouverte ?
    3. Calculer une valeur approchée à $10^{-3}$ près de la quantité de chlore le 15$^{\text{ième}}$ jour juste après l'ajout de chlore.
Partie B

Au fil du temps, la quantité de chlore évolue. On note $d_n$ l'écart de quantité de chlore d'un jour à l'autre en grammes. Pour tout entier naturel $n$, on a $d_n = u_{n+1}- u_n$.

    1. Calculer $d_0, d_1$ et $ d_2$. On donnera une valeur exacte.
    2. Justifier que $d_0, d_1$ et $ d_2$ semblent être les termes d'une suite géométrique.
  1. Vérifier que $u_{n+1}- u_n = -0,25u_n + 570$.
  2. On admet que pour tout entier naturel $n$, on a $d_{n+1}= 0, 75d_n$.
    1. Justifier que $d_n = 382,5 \times 0, 75^n$.
    2. En déduire que pour tout entier naturel $n$, on a $u_n = 2280 - 1530\times 0,75^n$.
    3. Déterminer la limite de la suite $(u_n)$. Interpréter le résultat trouvé.

Correction de l'exercice 2 (6 points)


Suites

Un centre de vacances possède une piscine de $600\quad m^3$ soit $600\quad 000$ litres. L'eau du bassin contient du chlore qui joue le rôle de désinfectant. Toutefois le chlore se dégrade et 25% de celui-ci disparaît chaque jour, en particulier sous l'effet des ultra-violets et de l'évaporation. Le 31 mai à 9 h, le responsable analyse l'eau du bassin à l'aide d'un kit distribué par un magasin spécialisé.
Le taux de chlore disponible dans l'eau est alors de 1,25 mg/L (milligrammes par litre).

Document
Réglementation des piscines publiques

$$\begin{array}{|c|c|c|} \hline \text{Paramètres contrôlés} & \text{Seuils de qualité réglementaire} &\text{Incidences sur la qualité de l'eau} \\ \hline & \text{Au minimum 2 mg/L} & < \text{2 mg/L : sous chloration}\\ &&\text{Risque de prolifération}\\ && \text{bactérienne dans l eau }\\ \hline \text{Présence de Chlore }& \text{ Au maximum 4 mg/L }&\text{ > 4 mg/L : surchloration }\\ &&\text{Irritation de la peau}\\ \hline \end{array} $$

Source : Agence Régionale de Santé  

A partir du 1$^{\text{er}}$ juin pour compenser la perte en chlore, la personne responsable de l'entretien ajoute, chaque matin à 9 h, 570 g de chlore dans la piscine.
Pour le bien-être et la sécurité des usagers, le responsable souhaite savoir si cet apport journalier en chlore permettra de maintenir une eau qui respecte la réglementation donnée par l' Agence Régionale de Santé pour les piscines publiques.

Partie A
  1. Pour tout entier naturel $n$ on note $u_n$ la quantité de chlore disponible, exprimée en grammes, présente dans l'eau du bassin le $n^{\text{ième}}$ jour suivant le jour de l'analyse, immédiatement après l'ajout de chlore. Ainsi $u_0$ est la quantité de chlore le 31 mai à 9 h et $u_1$ est la quantité de chlore le 1$^{\text{er}}$ juin à 9 h après l'ajout de chlore.
    1. Montrer que la quantité de chlore, en grammes, présente dans l'eau du bassin le 31 mai à 9h est $u_0 = 750$.
      Au regard des recommandations de l'agence régionale de santé, le responsable pouvait-il donner l'accès à la piscine le 31 mai?
    2. La quantité de chlore en milligrammes , présente dans l'eau du bassin le 31 mai à 9h est : \begin{align*} u_0&=1,25 \times 600\; 000\; \text{mg}\\ &=\dfrac{1,25 \times 600\; 000}{1\; 000}\; \text{g}\\ &= 750 \; \text{g} \end{align*} Ainsi $u_0 = 750$.
      Au regard des recommandations de l'agence régionale de santé, le responsable ne pouvait pas donner l'accès à la piscine le 31 mai car le taux de 1,25 mg/L est inférieur à 2 mg/L .
      On est dans un cas de sous chloration avec un risque de prolifération bactérienne.
    3. Montrer que $u_1 = 1132,5$.
    4. \begin{align*} u_1&=0,75\times u_0+570\\ &=1132,5 \end{align*}
    5. Justifier que pour tout entier naturel $n, u_{n+1} = 0, 75u_n+ 570$
    6. On rappelle qu'une baisse de $t $ % revient à multiplier par $\left (1-\dfrac{t}{100}\right )$; ici la baisse de 25\% revient à multiplier par $\left (1-\dfrac{25}{100}\right )=0,75$
      $u_{n+1}=\underbrace{ 0,75 u_n}_{ \text{ Baisse de 25 %}}+\underbrace{570}_{\text{ Ajout quotidien de 570 g }}$
    7. La suite $(u_n)$ est-elle géométrique ?
    8. \begin{align*} u_0&=750&\\ u_1&=1132,5&\\ u_2&=0,75\times 1132,5+570&\\ &=1479,375&\\ \dfrac{u_1}{u_0}&=1,51&\dfrac{u_2}{u_1} =1,3063\\ \end{align*} Comme $\dfrac{u_1}{u_0}\neq \dfrac{u_2}{u_1}$, la suite $(u_n)$ n'est pas géométrique.
  2. Soit l'algorithme ci-dessous : $$\begin{array}{|l |l |}\hline \text{Variables } & \\ &u : \text{un nombre réel }\\ &N : \text{un nombre entier naturel }\\ &k : \text{un nombre entier naturel }\\ \text{ Initialisation :}&\\ & \text{Saisir la valeur de } N \\ \text{ Initialisation :}&\\ & u \text{ prend la valeur } 750 \\ \text{ Traitement :}&\\ &\text{ Pour } k \text{ allant de } 1 \text{ à } N\\ &\hspace{0,5cm} u\text{ prend la valeur } 0,75 u +570\\ &\text{ Fin du Pour }\\ \text{ Sortie : }& \text{ Afficher } u \\ \hline \end{array}$$
    1. Quel est le rôle de cet algorithme ?
    2. Cet algorithme permet de calculer les termes successifs de $(u_n)$ pour $n$ allant de 1 à $N$ et d'afficher en sortie $u_N$
    3. Recopier et compléter le tableau suivant, par des valeurs exactes, en exécutant cet algorithme \og pas à pas »pour $N=3$. $$\begin{array}{|c|c|c|c|c|} \hline \text{Variables } & \text{ Initialisation }& \text{ Etape 1 } & \text{ Etape 2 } & \text{ Etape 3 } \\ \hline u & 750 &1132,5& & \\ \hline \end{array} $$ Au regard des recommandations de l'agence régionale de santé, au bout de combien de jours la piscine peut-elle être ouverte ?
    4. $$\begin{array}{|c|c|c|c|c|} \hline \text{Variables } & \text{ Initialisation }& \text{ Etape 1 } & \text{ Etape 2 } & \text{ Etape 3 } \\ \hline u & 750 &1132,5& 1419,375 & 1634,53125 \\ \hline \end{array} $$
      Au regard des recommandations de l'agence régionale de santé, au bout de combien de jours la piscine peut-elle être ouverte ?
      Au bout d'un jour le taux de chlore disponible dans l'eau est alors de $\dfrac{1132,5}{600}\approx 1,89$ mg/L . Au bout de deux jours le taux de chlore disponible dans l'eau est alors de $\dfrac{1419,375,5}{600}\approx 2,37$ mg/L .
      La piscine pourra donc être ouverte au bout de deux jours.
    5. Calculer une valeur approchée à $10^{-3}$ près de la quantité de chlore le 15$^{\text{ième}}$ jour juste après l'ajout de chlore.
    6. On utilise l'algorithme et on obtient $u_{15}\approx 2259, 554$
Partie B

Au fil du temps, la quantité de chlore évolue. On note $d_n$ l'écart de quantité de chlore d'un jour à l'autre en grammes. Pour tout entier naturel $n$, on a $d_n = u_{n+1}- u_n$.

    1. Calculer $d_0, d_1$ et $ d_2$. On donnera une valeur exacte.
    2. \begin{align*} d_0 &= u_{1}- u_0&=382,5\\ d_1 &= u_{2}- u_1&=286,875\\ d_2 &= u_{2}- u_1&=215,15625\\ \end{align*}
    3. Justifier que $d_0, d_1$ et $ d_2$ semblent être les termes d'une suite géométrique.
    4. On a $\dfrac{d_1}{d_0}=0,75$ et $\dfrac{d_2}{d_1}=0,75$,
      $d_0, d_1$ et $ d_2$ semblent être les termes d'une suite géométrique de raison $0,75$.
  1. Vérifier que $u_{n+1}- u_n = -0,25u_n + 570$.
  2. \begin{align*} u_{n+1}- u_n&=0, 75u_n+ 570-u_n\\ &= -0, 25u_n+ 570 \end{align*}
  3. On admet que pour tout entier naturel $n$, on a $d_{n+1}= 0, 75d_n$.
    1. Justifier que $d_n = 382,5 \times 0, 75^n$.
    2. Ayant pour tout entier naturel $n$, on a $d_{n+1}= 0, 75d_n$, la suite $(d_n)$ est géométrique de raison $q=0,75$, de premier termerme $d_0=382,5$,
      \begin{align*} d_n&=q^n \times d_0\\ &= 382,5 \times 0, 75^n \end{align*}
    3. En déduire que pour tout entier naturel $n$, on a $u_n = 2280 - 1530\times 0,75^n$.
    4. $u_{n+1}- u_n = -0,25u_n + 570$ et $d_n = u_{n+1}- u_n$, on adonc
      $d_n = -0,25u_n + 570$, \begin{align*} d_n = -0,25u_n + 570&\iff -0,25 u_n=d_n-570\\ &\iff -4\times -0,25 u_n=-4\times \left (d_n-570\right )\\ &\iff u_n= -4d_n +2280\\ &\iff u_n=-4\times 382,5 \times 0, 75^n +2280\\ &\iff u_n= 2280 - 1530\times 0,75^n \end{align*}
    5. Déterminer la limite de la suite $(u_n)$. Interpréter le résultat trouvé.
    6. Comme $01< 0,75 < 1$ on déduit $\lim\limits_{n\to +\infty}0,75^n=0$ et donc $\lim\limits_{n\to +\infty}u_n=2280$
      Le taux de chlore au bout d'un grand nombre de jours sera très proche de $\dfrac{2280}{600}=3,8$.
      Ce taux étant compris entre 2 mg/L et 4 mg/L, la situation sera stable ! On pourra donc laisser la piscine ouverte ...

 


Exercice 3 4 points


Fonctions

Quand l'oreille humaine est soumise à une intensité acoustique, exprimée en watts par mètre carré (W/m$^2$), le niveau sonore du bruit responsable de cette intensité acoustique est exprimé en décibels (dB).

Document
Echelle de bruit

\[\begin{array}{|c|c|c|c|}\hline \text{Sources sonores} &\text{Intensité} &\text{Niveau} & \text{Sensation auditive} \\ &\text{acoustique} &\text{sonore} &\\ &\text{(W/m}^2\text{ )} &\text{arrondi} & \\ & &\text{éventuellement}&\\ & &\text{à l'unité} &\\\hline \text{Décollage de la Fusée Ariane}& 10^ 6 &180 &\text{Exige une protection spéciale} \\ \hline \text{Turboréacteur} & 10^2 &140 &\text{Exige une protection spéciale} \\ \hline \text{Course de Formule 1} & 10 &130 &\text{Exige une protection spéciale} \\ \hline \text{Avion au décollage} & 1 &120 &\text{Seuil de douleur} \\ \hline \text{Concert et discothèque} & 10^{-1} &110 &\text{Très difficilement supportable} \\ \hline \text{Baladeur à puissance}& 10^{-2} &100 & \text{Très difficilement supportable} \\ \text{ maximum } & & &\\ \hline \text{ Moto} & 10^{-5} & 70 &\text{Pénible à entendre} \\ \hline \text{ Voiture au ralenti}&10^{-7} & 50 &\text{Bruit courant}\\ \hline \text{Seuil d'audibilité} & 10^{-12} & 0,08 &\text{Silence anormal} \\ \hline \end{array}\]

  1. D'après le tableau, lorsque l'intensité acoustique est multipliée par 10, quelle semble être l'augmentation du niveau sonore ?
  2. La relation liant l'intensité acoustique $x$ où $x$ appartient à l'intervalle $\left [10^ {-12} ; 10^6\right ]$ et le niveau sonore est donnée par : $$f (x) =\dfrac{10}{\ln 10} \times \ln(x) + 120.$$ On pourra prendre $\dfrac{10}{\ln 10} \approx 4,34$.
    1. Vérifier la conjecture émise à la question 1.
    2. Quel serait le niveau sonore de deux motos ?
  3. Pour éviter tout risque sur la santé, le port d'un casque de protection acoustique est donc conseillé au delà de 85 dB. Déterminer l'intensité acoustique à partir de laquelle le port d'un tel casque est conseillé.

Correction de l'exercice 3 (4 points)


Fonctions

Quand l'oreille humaine est soumise à une intensité acoustique, exprimée en watts par mètre carré (W/m$^2$), le niveau sonore du bruit responsable de cette intensité acoustique est exprimé en décibels (dB).

Document
Echelle de bruit

\[\begin{array}{|c|c|c|c|}\hline \text{Sources sonores} &\text{Intensité} &\text{Niveau} & \text{Sensation auditive} \\ &\text{acoustique} &\text{sonore} &\\ &\text{(W/m}^2\text{ )} &\text{arrondi} & \\ & &\text{éventuellement}&\\ & &\text{à l'unité} &\\\hline \text{Décollage de la Fusée Ariane}& 10^ 6 &180 &\text{Exige une protection spéciale} \\ \hline \text{Turboréacteur} & 10^2 &140 &\text{Exige une protection spéciale} \\ \hline \text{Course de Formule 1} & 10 &130 &\text{Exige une protection spéciale} \\ \hline \text{Avion au décollage} & 1 &120 &\text{Seuil de douleur} \\ \hline \text{Concert et discothèque} & 10^{-1} &110 &\text{Très difficilement supportable} \\ \hline \text{Baladeur à puissance}& 10^{-2} &100 & \text{Très difficilement supportable} \\ \text{ maximum } & & &\\ \hline \text{ Moto} & 10^{-5} & 70 &\text{Pénible à entendre} \\ \hline \text{ Voiture au ralenti}&10^{-7} & 50 &\text{Bruit courant}\\ \hline \text{Seuil d'audibilité} & 10^{-12} & 0,08 &\text{Silence anormal} \\ \hline \end{array}\]

  1. D'après le tableau, lorsque l'intensité acoustique est multipliée par 10, quelle semble être l'augmentation du niveau sonore ?
  2. D'après le tableau, lorsque l'intensité acoustique est multipliée par 10, on semble ajouter 10 dB au niveau sonore.
  3. La relation liant l'intensité acoustique $x$ où $x$ appartient à l'intervalle $\left [10^ {-12} ; 10^6\right ]$ et le niveau sonore est donnée par : $$f (x) =\dfrac{10}{\ln 10} \times \ln(x) + 120.$$ On pourra prendre $\dfrac{10}{\ln 10} \approx 4,34$.
    1. Vérifier la conjecture émise à la question 1.
    2. \begin{align*} f(10x) & = \dfrac{10}{\ln 10} \times \ln(10 x) + 120& \text{ On a multiplié par 10 ... }\\ &=\dfrac{10}{\ln 10} \times \left ( \ln(10) +\ln ( x) \right )+ 120& \\ &=\dfrac{10}{\ln 10} \times \left ( \ln 10 +\ln ( x) \right )+ 120& \\ &=\dfrac{10}{\ln 10} \times \ln 10 + \dfrac{10}{\ln 10} \times \ln x + 120 \\ &= 10 + \underbrace{\dfrac{10}{\ln 10} \times \ln x + 120 }_{f(x)}\\ &= f(x)+ 10 \end{align*}
    3. Quel serait le niveau sonore de deux motos ?
    4. Pour calculer le le niveau sonore de deux motos on calcule $f\left ( 2\times 10^{-5}\right )$ \begin{align*} f\left ( 2\times 10^{-5}\right ) & = \dfrac{10}{\ln 10} \times \ln\left ( 2\times 10^{-5}\right ) + 120& \\ &=\dfrac{10}{\ln 10} \times \left ( \ln2 +\ln \left ( 10^{-5}\right ) \right )+ 120& \text{ car } \ln (a \times b)= \ln a +\ln b \\ &=\dfrac{10}{\ln 10} \times \ln 2 + \dfrac{10}{\ln 10} \times \left ( 10^{-5}\right ) + 120 \\ &= \dfrac{10\ln 2}{\ln 10} + \underbrace{ \dfrac{10}{\ln 10} \times \left ( 10^{-5}\right ) + 120 }_{f\left ( 10^{-5}\right )}\\ &= \dfrac{10\ln 2}{\ln 10} +70 \\ &\approx 73 \end{align*} Remarque: on peut bien sûr vérifier ce calcul à l'aide d'une calculatrice !
      Le niveau sonore de deux motos est environ 73 dB.
  4. Pour éviter tout risque sur la santé, le port d'un casque de protection acoustique est donc conseillé au delà de 85 dB. Déterminer l'intensité acoustique à partir de laquelle le port d'un tel casque est conseillé.
  5. On résout $f(x) < 85$ \begin{align*} f(x) < 85 & \iff \dfrac{10}{\ln 10} \times \ln(x) + 120 < 85 & \\ & \iff \dfrac{10}{\ln 10} \times \ln(x) < -35 & \\ & \iff \ln x < -\dfrac{ 35 \ln 10 }{10} & \text{ car } \dfrac{10}{\ln 10} \approx 4,34 > 0 \\ &\iff x < \text{e}^{ -\frac{35 \ln 10 }{10} } & \text{ car } x \mapsto \text{e}^x \text{ est st. croissante sur } \mathbb R \\ &\iff x < \left (\text{e}^{ \ln 10 } \right )^{-\frac{35}{10}}& \\ &\iff x < 10^{-3,5} \end{align*} Or $10^{-3,5}\approx 3,2 \times 10^{-4}$
    L'intensité acoustique à partir de laquelle le port d'un tel casque est conseillé est environ $3,2 \times 10^{-4}$ W/m$^2$. On résout $f(x) < 85$
    On utilise la calculatrice :
    Capturer1 Capturer2
    Capturer1 Capturer2
    Capturer1  

Exercice 4 6 points


Probabilités

Les parties A et B sont indépendantes.
Un pont levant enjambant un canal peu fréquenté est constitué d'un tablier qui, une fois relevé, permet le passage de bateaux de différentes tailles.
pont
$$\begin{array}{|l |} \hline \text{Hauteur du tablier en position haute : 7 mètres}\\ \text{Longueur du tablier : 30 mètres}\\ \text{Temps de montée du tablier : 2 minutes}\\ \text{Temps en position haute du tablier (hors incident) : 8 minutes}\\ \text{Temps de descente du tablier : 2 minutes}\\ \hline \end{array} $$

Partie A - Sur la route

Un automobiliste se présente devant le pont. Le tablier du pont est en position haute. On s'intéresse ici au temps d'attente D, exprimé en minutes, de l'automobiliste avant qu'il puisse franchir le canal, pont baissé (hors incident).

 

  1. Combien de temps l'automobiliste attend-il au minimum ? au maximum?
  2. On admet que le temps d'attente, en minutes, de l'automobiliste pour franchir le pont est une variable aléatoire D qui suit la loi uniforme sur l'intervalle [2 ;10].
    Déterminer l'espérance E(D) de la variable aléatoire D et interpréter le résultat dans le contexte.
  3. Calculer la probabilité que le temps d'attente de l'automobiliste ne dépasse pas 5 minutes.
Partie B - Sur l'eau

Dans cette partie les résultats demandés seront arrondis à $10^ {-2}$ près.
Lorsqu'un bateau est passé, le tablier du pont revient en position basse. Le temps, exprimé en heures, avant que le bateau suivant se présente devant le pont est une variable aléatoire T qui suit la loi exponentielle de paramètre $\lambda = 0,05$. Ce temps est appelé temps de latence.

  1. Déterminer l'espérance E(T) de la variable aléatoire T et interpréter le résultat dans le contexte.
  2. On considère la fonction $f$ définie sur $[0 , +\infty[$ par $f (x) = 0,05 \text{e}^{-0,05 x}$.
    1. Montrer que la fonction $F$ définie sur$[0 , +\infty[$ par $F(x) =-\text{e}^{-0,05 x}$ est une primitive de $ f$ .
    2. On rappelle que pour tout nombre réel $t$ de $[0 , +\infty[ , P(T \leq t) = \displaystyle\int_0 ^t f(x)\; dx$.
      Démontrer que $P(T \leq t) =1- \text{e}^{-0,05 t}$.
    1. Calculer la probabilité que le temps de latence soit inférieur à une demi-journée, soit 12 heures.
    2. Calculer la probabilité que le temps de latence soit supérieur à un jour.
    3. Calculer $P(12\leq T\leq 24)$.

 


Exercice 4 6 points


Probabilités

Les parties A et B sont indépendantes.
Un pont levant enjambant un canal peu fréquenté est constitué d'un tablier qui, une fois relevé, permet le passage de bateaux de différentes tailles.
pont
$$\begin{array}{|l |} \hline \text{Hauteur du tablier en position haute : 7 mètres}\\ \text{Longueur du tablier : 30 mètres}\\ \text{Temps de montée du tablier : 2 minutes}\\ \text{Temps en position haute du tablier (hors incident) : 8 minutes}\\ \text{Temps de descente du tablier : 2 minutes}\\ \hline \end{array} $$

Partie A - Sur la route

Un automobiliste se présente devant le pont. Le tablier du pont est en position haute. On s'intéresse ici au temps d'attente D, exprimé en minutes, de l'automobiliste avant qu'il puisse franchir le canal, pont baissé (hors incident).

 

  1. Combien de temps l'automobiliste attend-il au minimum ? au maximum?
  2. Le tablier du pont étant en position haute, l'automobiliste attendra au minimum 2 minutes (Temps de montée du tablier )
    Au maximum, l'automobiliste attendra 10 minutes (Temps en position haute du tablier (hors incident) : 8 minutes +Temps de descente du tablier + 2 minutes)
  3. On admet que le temps d'attente, en minutes, de l'automobiliste pour franchir le pont est une variable aléatoire D qui suit la loi uniforme sur l'intervalle [2 ;10].
    Déterminer l'espérance E(D) de la variable aléatoire D et interpréter le résultat dans le contexte.
  4. Rappel : Si D suit la loi uniforme sur $[a; b]$ alors l'espérance E(D)=$\dfrac{a+b}{2}$ \begin{align*} E(D)&=\dfrac{2+10}{2}\\ &=6 \end{align*}
    Le temps d'attente moyen, en minutes, de l'automobiliste pour franchir le pont est de 6 minutes .
  5. Calculer la probabilité que le temps d'attente de l'automobiliste ne dépasse pas 5 minutes.
  6. On calcule ici $P(D\leq 5)$ \begin{align*} P(D\leq 5)&=\dfrac{\text{longueur } [ 2~; ~5] }{\text{longueur } [ 2~; ~8]}\\ &= \dfrac{5-2}{10-2}\\ &=\dfrac{3}{8} \end{align*} La probabilité que le temps d'attente de l'automobiliste ne dépasse pas 5 minutes est 0,375.
Partie B - Sur l'eau

Dans cette partie les résultats demandés seront arrondis à $10^ {-2}$ près.
Lorsqu'un bateau est passé, le tablier du pont revient en position basse. Le temps, exprimé en heures, avant que le bateau suivant se présente devant le pont est une variable aléatoire T qui suit la loi exponentielle de paramètre $\lambda = 0,05$. Ce temps est appelé temps de latence.

  1. Déterminer l'espérance E(T) de la variable aléatoire T et interpréter le résultat dans le contexte.
  2. T suit une loi exponentielle donc son espérance mathématique vaut : \begin{align*} E(T)&=\dfrac{1}{\lambda}\\ &=\dfrac{1}{0,05}\\ & = 20 \end{align*} Le temps moyen avant que le bateau suivant se présente devant le pont est de 20 heures.
  3. On considère la fonction $f$ définie sur $[0 , +\infty[$ par $f (x) = 0,05 \text{e}^{-0,05 x}$.
    1. Montrer que la fonction $F$ définie sur$[0 , +\infty[$ par $F(x) =-\text{e}^{-0,05 x}$ est une primitive de $ f$ .
    2. Il suffit de vérifier que la dérivée de $F$ est $f$ \begin{align*} F'(x)&=-\left (-0,05\text{e}^{-0,05 x}\right )\\ &=0,05\text{e}^{-0,05 x}\\ &=f(x) \end{align*} $F$ définie sur$[0 , +\infty[$ par $F(x) =-\text{e}^{-0,05 x}$ est donc une primitive de $ f$ .
    3. On rappelle que pour tout nombre réel $t$ de $[0 , +\infty[ , P(T \leq t) = \displaystyle\int_0 ^t f(x)\; dx$.
      Démontrer que $P(T \leq t) =1- \text{e}^{-0,05 t}$.
    4. \begin{align*} P(T \leq t) & = \displaystyle\int_0 ^t f(x)\; dx\\ &=F(t)-F(0)\\ &=-\text{e}^{-0,05 t}-\left (-\text{e}^{0}\right )\\ &=1- \text{e}^{-0,05 t} \end{align*}
    1. Calculer la probabilité que le temps de latence soit inférieur à une demi-journée, soit 12 heures.
    2. \begin{align*} P(T \leq 12) & =1- \text{e}^{-0,05 \times 12}\\ &= 1- \text{e}^{-0,6}\\ &\approx 0,45 \end{align*}
    3. Calculer la probabilité que le temps de latence soit supérieur à un jour.
    4. On calcule donc $P(T\geq 24)$ \begin{align*} P(T\geq 24)&=1-P(T<24)\\ &=1-P(T\leq 24)\\ &=1-\left ( 1- \text{e}^{-0,05 \times 24}\right )\\ &= \text{e}^{-1,2}\\ & \approx 0,30 \end{align*}
    5. Calculer $P(12\leq T\leq 24)$.
    6. \begin{align*} P(12\leq T\leq 24) & = \displaystyle\int_{12} ^{24} f(x)\; dx\\ &=F(24)-F(12)\\ &=-\text{e}^{-0,05 \times 24}-\left (-\text{e}^{-0,05 \times 12}\right )\\ &=\text{e}^{-0,6}-\text{e}^{-1,2}\\ &\approx 0,25 \end{align*}

 

  • Vues: 11238

Rechercher